SlideShare uma empresa Scribd logo
1 de 46
Baixar para ler offline
44
RAZÕES PORQUE O
SABADO NÃO COMEÇA
AO POR-DO-SOL
Segundo o peso da evidência nas Escrituras, um dia é de apenas 12
horas de duração e começa ao nascer do sol e termina ao pôr do sol. A
noite é dividida por quatro relógios com início ao pôr do sol e
terminando ao amanhecer. A data do calendário completo é 24 horas
de comprimento e não é sinônimo de um dia. Em contraste uma data
do calendário civil plena inclui todos os componentes de ambos o dia ea
noite, e começa ao amanhecer e não à meia-noite ou pôr do sol. Um dia
é apenas a primeira metade de uma data do calendário lunar completo,
enquanto a noite é a segunda metade. O dia ea noite ocupar porções
opostas de tempo em uma data do calendário lunar completo. Dia e
noite NUNCA sobrepõem ou ocupar o mesmo espaço de tempo.
Para as pessoas muito tempo, boas e honestas têm se contentado em
seguir fábulas judaicas e romanas (Tito 1:14) para quando o dia
começa que não são suportados pelo peso da evidência nas Escrituras.
No entanto, ao longo de toda a verdade foi lá para a tomada, com um
pouco de esforço para reunir todas as provas do original hebraico ou
palavras gregas.
O simples facto de a esta hora tardia esta unidade de tempo está sendo
revelado, deve dizer-nos o nosso Messias está voltando em breve para
seus fiéis, que será totalmente restaurada para a cronometragem de
Seu reino. A seguir, a verdade presente é projetado para equipar fiel
de Yahuah para as batalhas mais verdade que temos pela frente.
Pergunta 1: De acordo com Gênesis 1:1-5, qual foi a primeira coisa que
o Criador convocado à existência?
No começo [Elohiym] criou os céus ea terra. A terra era sem forma e
vazia, e havia trevas sobre a face do abismo. E o Espírito de [Elohiym]
pairava sobre a face das águas. Então [Elohiym] disse: "Haja luz", e
houve luz. E [Elohiym] viu a luz que era bom, e [Elohiym] separação
entre a luz e as trevas. [Elohiym] chamou à luz Dia e às trevas chamou
Noite. Gênesis 1:1-5 NVI
Resposta: A primeira coisa que Ele criou era "luz".
Pergunta 2: Qual foi a primeira coisa que o Criador chamado?
No começo [Elohiym] criou os céus ea terra. A terra era sem forma e
vazia, e havia trevas sobre a face do abismo. E o Espírito de [Elohiym]
pairava sobre a face das águas. Então [Elohiym] disse: "Haja luz", e
houve luz. E [Elohiym] viu a luz, que era bom, e [Elohiym] separação
entre a luz e as trevas. [Elohiym] chamou à luz dia, e às trevas chamou
Noite. Gênesis 1:1-5 NVI
Resposta: Luz foi a primeira coisa que o Criador nomeado. Ele
chamou o "dia". Luz
Questão 3: Qual foi a segunda coisa que o Criador chamado?
No começo [Elohiym] criou os céus ea terra. A terra era sem forma e
vazia, e havia trevas sobre a face do abismo. E o Espírito de [Elohiym]
pairava sobre a face das águas. Então [Elohiym] disse: "Haja luz", e
houve luz. E [Elohiym] viu a luz, que era bom, e [Elohiym] separação
entre a luz e as trevas. [Elohiym] chamou à luz dia, e às trevas chamou
Noite. Gênesis 1:1-5 NVI
Resposta: A escuridão era a segunda coisa que o Criador nomeado. Ele
chamou das trevas "noite".
Pergunta 4: De acordo com Gênesis 1:1-5, quando foi que o Criador
fazer a escuridão?
No começo [Elohiym] criou os céus ea terra a terra era sem forma e
vazia;. E havia trevas sobre a face do abismo. E o Espírito de
[Elohiym] pairava sobre a face das águas. Então [Elohiym] disse:
"Haja luz", e houve luz. E [Elohiym] viu a luz, que era bom, e
[Elohiym] separação entre a luz e as trevas. [Elohiym] chamou à luz
dia, e às trevas chamou Noite. Gênesis 1:1-5 NVI
Resposta: Por mais surpreendente que isso possa ser para alguns, o
Criador não fazer ou criar a escuridão. Como definido acima, a
escuridão é a ausência de luz. A escuridão é a ausência da força
criadora. Para fazer uma distinção entre a escuridão ea luz do criador
separados deles, o que significa que eles vão sempre atuar em
diferentes períodos de tempo e nunca se sobrepõem. Ele, então, selou
sua declaração de que Ele os separou quando Ele deu-lhes os seus
nomes. Dia e noite, não são termos sinônimos, mas sim que eles são
opostos.
Pergunta 5: Uma vez que a escuridão é a ausência de luz, então o que é
a noite?
Resposta: a noite é a ausência do dia, tal como definido por Genesis
1:1-5.
Questão 6: De acordo com o Novo Testamento inteiro, a luz do dia é
dividido em 12 unidades básicas de tempo. O que são chamadas essas
unidades?
E quando veio aqueles que foram contratados cerca da hora undécima,
receberam um denário cada um. Mateus 20:9-10 NVI
Jesus respondeu: "Não são doze as horas do dia? Se alguém andar de
dia, não tropeça, porque vê a luz deste mundo. 10 Mas, se alguém
andar de noite, tropeça, porque a luz é não nele ". João 11:9-11 NVI
Resposta: Essas unidades são chamadas de "horas". A luz do dia, de
sol a sol, é composta por doze horas. Horas foram contados pelas
sombras em um relógio de sol. Mesmo que os dias de verão são mais
longos do que os dias de inverno, ele sempre aparece em um relógio de
sol como 12 horas.
Pergunta 7: Um relógio funciona somente durante as horas de sol a sol,
então o que foi usado para contar as unidades de tempo durante a
escuridão da noite?
Resposta: É os celestes estrelas / planetas nos céus acima que
determinar o comprimento das quatro vigílias da noite (Marcos 13:35).
Noite Watch - Esta primeira vigília da noite começa ao pôr do sol e
ocupou o período conhecido como entardecer ou crepúsculo, a mistura
de luz e trevas, e termina na escuridão completa. Mateus 13:35
Meia-noite Watch - Esta segunda vigília da noite começa às trevas
completo e termina precisamente à meia-noite. Era a posição da lua e
as estrelas que declararam verdadeiro meia-noite. Lucas 0:38
Galo cantando Watch - Esta terceira vigília da noite começa à meia-
noite e termina na primeira luz da aurora ou crepúsculo da manhã.
Além disso, este foi o período em que era característico de quando o
galo começou a cantar. Lucas 0:38
Manhã Watch - Esta quarta vigília da noite, começa na primeira luz
visível no céu do leste, o tempo que o galo começou a cantar, o que
também é conhecido como aurora ou crepúsculo da manhã, a mistura
de luz e escuridão. A manhã termina Assista ao nascer do sol. Mateus
14:25
Pergunta 8: Existe uma aplicação secundária, o que tornaria a divisão
do "dia ea" noite ", uma lei espiritual do Criador.
Resposta: De acordo com a Escritura, a noite foi dividida em quatro
relógios. A lua, juntamente com a posição das estrelas, foi o guia para
conhecer cada quarta parte da noite. Observe como esses parâmetros
da noite também definem os limites do "dia". Não pode haver
sobreposição. A noite termina como o novo dia começa ao amanhecer.
O termo "turno da noite" nasceu da idéia de ver os primeiros raios de
um novo dia ao amanhecer. O foco estava sempre na luz, procurando a
luz, sempre buscando a luz. Desta forma, uma ilustração viva foi
criado que expressa os lugares valor Yahuah sobre a luz do dia, em
contraste com a escuridão da noite.Porque todo aquele que faz o mal
odeia a luz e não vem para a luz, para que as suas obras não sejam
reprovadas. Mas quem pratica a verdade vem para a luz que as suas
obras sejam manifestas, porque são feitas em [Elohiym]. João 3:20-21
Mas, se os teus olhos forem maus, todo o teu corpo será tenebroso.
Portanto, caso a luz que em ti há são trevas, quão grandes serão tais
trevas! Ninguém pode servir a dois senhores, porque ou há de odiar a
um e amar o outro, ou se dedicará a um e desprezará o outro. Não
podeis servir a [Elohiym] ea Mamom. Mateus 6:23-24
Porque todos vós sois filhos da luz e filhos do dia: nós não somos da
noite nem das trevas. Portanto, não durmamos, como os demais, mas
vigiemos e sejamos sóbrios. 1 Tessalonicenses 5:5-6
Mais uma vez, um novo mandamento que vos escrevo, o qual é
verdadeiro nele e em vós, porque as trevas se vão dissipando, ea
verdadeira luz já brilha. Aquele que diz que está na luz, e odeia a seu
irmão, está nas trevas até agora. Aquele que ama a seu irmão
permanece na luz, e não há nenhuma causa para tropeço nele. Mas
aquele que odeia a seu irmão está nas trevas e anda nas trevas, e não
sabe para onde vai, porque as trevas lhe cegaram os olhos. 1 João 2:8-
11 NVI
Pergunta 9: Se Yahuah dá o sol para luz do dia, e as estrelas para luz
da noite, então como pode o dia começa ao pôr do sol ou anoitecer, à
meia-noite, de manhã crepúsculo antes do nascer do sol, ou ao meio-
dia? Há pessoas ao redor do mundo que reconheceram esses períodos
como o início de seu "dia" e, especificamente, em nome de seu santo
dia.
Resposta: De acordo com Jeremias 31:35, quando a primeira estrela é
testemunhado no céu, podemos saber que a "noite" já começou.
Enquanto o próprio sol é visível, o que significa nascer ao pôr do sol, é
"dia em que o sol está no poder."
Assim diz Yahuah, que dá o sol para luz do dia, as ordenanças da lua e
das estrelas para luz da noite. "Se estas ordenanças de diante de mim,
diz Yahuah, então a semente de Israel deixam também de ser uma
nação diante de mim para sempre." Jeremias 31:35-36 NTLH
A partir deste versículo podemos adquirir os seguintes fatos:
Quando o versículo acima fala das estrelas que está falando de estrelas
visíveis.
Quando o versículo acima fala do sol o mesmo princípio aplica-se, por
isso, fala de um sol visível.
Quando as estrelas estão presentes e visíveis, é "noite".
Quando o sol está presente e visível, é "dia".
O sol e as estrelas nunca estão visíveis ao mesmo tempo.
A lua visível também é falado, embora a segunda metade do mês a lua
nem sempre é visível à noite, durante as fases lunares diminuindo.
"Esta é a instrução direta pelo Criador, Yahuah, para a divisão do dia
e da noite.
Dia e noite estão aqui diferenciados especificamente pelo "sol" e as
"estrelas". Eles simplesmente não são visíveis ao mesmo tempo.
Quando o sol deixa de brilhar as estrelas começam o seu resplendor.
Por favor, note que não é apenas a luz que é dado para a parte que
compõe o "dia", mas afirma o "sol" é para luz do dia. Metade do mês
a lua é visível à noite e metade do mês, é visível a dia. Mas as estrelas
nunca variam sobre o seu resplendor. A palavra "decreto", aqui,
significa o " tempo determinado circuito do sol, a lua e as estrelas. " É
a lua e as estrelas que, especificamente, dar luz à noite. Ao pôr do sol
de um ou dois planetas / estrelas começam a ser visíveis, e uma ou duas
estrelas continuam a ser visível até o amanhecer.
Se o dia começou ao meio-dia, sol, ou meia-noite, ele faria com que o
sol, a lua e as estrelas para se sobrepor. Gênesis 1:14-18 define o
parâmetro de dia e noite com a simples declaração de que o sol estava
para governar o dia, ea lua e as estrelas foram para governar a noite.
Quanto mais claro o Pai Celestial poderia ser? Se o "dia" ocupou o
período desde o amanhecer até o final do entardecer, o "dia" também
estaria sobrepondo o brilho das estrelas ea lua para 1 ½ a 2 horas ao
anoitecer e, em seguida, novamente, pouco antes do nascer do sol. Nós
podemos saber com certeza que o período imediatamente antes do
nascer do sol ainda é considerado "noite" pelo Criador, bem como logo
após pôr do sol, porque há pelo menos um planeta / estrela visível
pouco antes do nascer do sol, e pelo menos um planeta / estrela visível
apenas após o ocaso.
Foi o Criador, mesmo quem esclareceu que a divisão entre "dia" e
"noite" se manifesta pelo sol visível, a lua e as estrelas. Quando o "dia"
é dividido corretamente, não haverá sobreposição de suas luzes
celestiais. Portanto, a única divisão possível do dia e da noite é de sol a
sol, já que não há sobreposição de luz do sol com a lua e as estrelas. No
momento em que os raios do sol radiantes desaparecer ao pôr do sol as
primeiras estrelas começam a aparecer em perfeita harmonia com a
divisão do dia e da noite, no Gênesis 1 e Êxodo conta 16. Consulte o
artigo, " estrelas visíveis definir os parâmetros de dia e noite . "
Pergunta 10: De acordo com Gênesis 1:5, quando é que uma noite
começa e termina?
[Elohiym] chamou à luz dia, e às trevas chamou Noite. Gênesis 1:1-5
NVI
Resposta: A noite começa ao pôr do sol como este é quando a escuridão
se mistura com a luz. A noite termina ao nascer do sol do amanhecer
seguinte mistura de luz e escuridão. Então noite ocupa a segunda
parcela de uma data do calendário lunar logo após pôr do sol, quando
as estrelas começam um a um, para tornar-se visivelmente presente.
Pergunta 11: Assim, se uma noite começa ao pôr do sol, ocupando todo
o período negro, quando é que um dia começa?
[Elohiym] chamou à luz dia, e às trevas chamou Noite. Gênesis 1:1-5
NVI
Resposta: O dia foi contada apenas por as horas do dia, e começou ao
amanhecer e terminou ao pôr do sol. Quando as pessoas declaram que
o "dia" começa ao pôr do sol, eles estão realmente dizendo que "a luz é
igual a escuridão."
Pergunta 12: De acordo com Gênesis 1:5, que é a definição dada para a
palavra "noite?"
[Elohiym] chamou à luz dia, e às trevas chamou Noite. Assim foi a
tarde ea manhã, o dia primeiro. Gênesis 01:05 NVI
Resposta: O Criador ordenou que a luz é igual a dia e que a escuridão
é igual a noite, mas agora somos confrontados com o termo "noite",
que não é esclarecido em Gênesis 1, como o que ela é. . Utilize dois ou
mais léxicos hebraicos como foi feito abaixo para saber os significados
da palavra hebraica ('Ereb # 6153).
Definição palavra "noite" (Ereb # 6153)
# 1689 Theological Wordbook do Antigo Testamento (TWOT)
corresponde a # 6153 na Concordância de Strong.
'Ereb: Noite, noite. Provavelmente desenvolvido a partir da expressão
", o pôr do sol, pôr do sol." Inclui: para entrar, para ir para baixo do
sol. . .
'Ereb é encontrado 131 vezes no AT. A frase "não havia uma noite e
havia uma manhã" ocorre seis vezes na narrativa da criação (Gênesis
1:5-31), delimitando os seis dias de atividade criativa divina. Esta frase
parece indicar que no antigo Israel um dia começou com nascer do sol.
Alguns sentiram esta em desacordo com a prática judaica de
considerar pôr do sol como o início do dia seguinte. Cassuto, depois de
lidar com os dados bíblicos eo costume judaico, conclui que não era
"apenas um sistema de tempo de computação: o dia é considerado
para começar no dia ... Este julgamento parece justificado no emprego
de" Ereb na legislação levítico respeitando a imundícia. Uma era
considerada impura por causa de certos atos "até a noite" (Levítico
11:24, mais de trinta vezes). Isto é, um era imundo para a duração do
dia. Teológico Wordbook do Antigo Testamento (TWOT) # 1689, p.
694.
Significa: anoitecer, noite, noite, pôr do sol.
No estudo palavra abaixo, mostra-se que o termo "noite" é igual ao
momento do pôr do sol, o início da escuridão da noite, também
conhecida como a "troca de crepúsculo." As "trocas de crepúsculo"
ocorrem durante a noite, tanto o amanhecer eo entardecer, marcando
o início eo fim do do Criador "noite".
Stro
ng #
King
James
Versio
n da
Bíblia
Hebra
ico
Strong Definições Brown-driver-
Briggs Hebrew
Lexicon
Novo
Significado
Comprehensive
Erros evidentes na tradução e adicionado # 9999 palavras são destacadas neste verde
Versículo 5
7121 E qara ' para chamar;endereço
pelonome; convidar;
pregar, proclamar;
pronunciar; publicar;
falar;
chamar, a chamar,
a recitar, para ler,
para gritar, para
proclamar,
proclamar; aonom
e, para dar nome
a, para
Dirigindo-lo
pelo nome,
chamado
ser nomeado, a ser
escolhido;
430 [Elohiy
m]
'Elohiy
m
nome plural de
[Elohiym], referindo-se
especificamente ao
supremo
[Elohiym];'Elohiym;
'Elohiym (plural)
governantes,
juízes; os divinos;
anjos; [Elohiym]
s; (significado
intensivo singular
plural) [Elohiym];
o único e
verdadeiro
[Elohiym];
Elohiym,
216 a luz 'Owr iluminação;
brilhante; luz;
a luz do dia, a luz
de luminares
celestes (a Lua, o
Sol, as
estrelas); luz
dodia; madrugada,
a luz da manhã.
a luz
3117 dia yowm de uma raiz não
utilizada significando
ser quente; um dia
(como as
horasquentes), sol a sol,
pôr do sol ao pôr do sol;
(Nota:. Pôr do sol a sol
não se pode aplicar
aqui, porque o
processo de criação foi
especificamente
concebido para
separar a luz das
trevas, o dia da noite, e
não incluir a noite do
dia)
dia; tempo;
ano;por dia, em
oposição
anoite, um dia de
trabalho, uma
divisão de tempo;
"Dia" em
oposição à
noite (as horas
quentes do
nascer ao pôr do
sol),
2822 ea
escurid
ão
choshe
k
escuro; trevas,trevas;
noite;
trevas, trevas; ea escuridão
7121 chama
do
qara para chamar;endereço
pelo nome; convidar;
pregar, proclamar;
pronunciar; publicar;
falar;
chamar, a chamar,
a recitar, para ler,
para gritar, para
proclamar,
proclamar; ao
nome, para dar
nome a, para ser
nomeado, a ser
Chamou
escolhido;
3915 noite. layil noite,
aescuridão,estação
noite; um toque longe
da luz; temporada noite.
Nota: Noite não podem
ser incluídos no termo
"dia", porque o
Criador está aqui
fazendo uma distinção
entre "dia e noite",
separando-os, Ele não
é colocá-los juntos.
noite (em
oposição ao dia)
Nota: Este Léxico
hebraico define
noite como
distintamente
separada do
"dia". Um
princípio
fundamental é
definido aqui em
pedra pelo
Criador, a Si
mesmo que o dia
ea noite devem
ser para sempre e
distintamente
separada.
"Noite", em
oposição ao
dia(o período
escuro do pôr ao
nascer do sol).
1961 e Hayah de existir, ou tornar-
se, vir apassar, farol,
completamente; seguir,
aconteceu;
ser, para se tornar,
a
acontecer, paraexi
stir, a acontecer,
para cair;
Depois veio
6153 a noite `Ereb
`Ereb
# 6154
-
Palavr
a -
Root
# 6148
`árabe
anoitecer; maré da
noite, noite,dia
mistura
trançar, intermix; se
misturam; dar penhores;
envolver, dar para ser de
segurança (como uma
espécie detroca);
pôr
do sol,noite, noite
;
# 6154 `Ereb - que
é a mesma palavra
hebraica: trama,
como misto;entrel
açados; material
de malha; mistura;
pessoas mistos;
sociedade de
economia mista.
a troca, para
envolver, dar em
penhor, para
assumir um
compromisso,
para garantir;
o
anoitecer(Crepú
sculo)
Palavr
a Raiz
-
# 6150
`árabe
através da idéia de
cobrir com textura;
sol; serescurecido;
para troca de
promessas;
ter comunhão
com;
1961 e hayah de existir, ou tornar-se,
vir a passar, farol,
completamente;seguir,a
conteceu;
ser, para se
tornar, a
acontecer, paraexi
stir, a acontecer,
para cair;
em seguida,
veio
1242 Pela
manhã
Boqer madrugada,manhã, rom
per do dia;
manhã, final da
noite, vinda daluz
do dia;vinda
donascer
do sol;início
do dia;alegria
figurativamente
brilhante depois
de uma noite de
angústia, o dia de
amanhã, no dia
seguinte, na
manhã seguinte;
o amanhecer,
(Crepúsculo)
3117 dia. yowm de uma raiz não
utilizada significando
ser quente; um dia
(como as
horasquentes), sol a sol,
pôr do sol ao pôr do sol;
(Nota:. Pôr do sol a sol
não se pode aplicar
aqui, porque o
processo de criação foi
especificamente
concebido para
separar a luz das
trevas, o dia da noite, e
não incluir a noite do
dia)
dia; tempo;
ano;por dia, em
oposição
anoite, um dia de
trabalho, uma
divisão de tempo;
dia (em
oposição a noite,
as horas quentes
do nascer ao pôr
do sol)
259 o
primeir
'Echad um numeral que
significa unificar; um,
um (número),
cada, cada, um
um.
o como
umprimeiro ordinal;
certo, apenas, uma
vez que, uma vez
por todas, em
primeiro lugar;
Dirigindo-lo pelo nome, Elohiym chamou à luz dia, ao contrário de
noite, e às trevas chamou noite, ao contrário do dia. Depois veio o
anoitecer (crepúsculo), depois veio o amanhecer (crepúsculo), um dia.
Êxodo 01:05
Pergunta 13: De acordo com a tabela de Gênesis 1:05 palavra estudo
acima, se o Criador dizer que a noite faz parte do dia ou da parte da
noite?
Resposta: De acordo com o estudo da palavra hebraica "noite" (Ereb #
6153) significa pôr do sol, noite, e ao anoitecer. Cada um destes são
equiparados a noite, desde a noite começa ao pôr do sol.
Pergunta 14: As modernas versões da Bíblia afirmar que "tarde ea
manhã" foram o primeiro dia, segundo dia, etc, isso não significa que
"a noite vem antes do amanhecer?"
Resposta: Na superfície este parece ser o caso, no entanto, uma rápida
olhada em qualquer concordância ou léxico proporcionará ampla
evidência de que duas palavras foram mal traduzidas. A palavra
hebraica em questão aqui é (hayah # 1961), o que significa: existir,
tornar-se, acontecer, vir a passar, siga, (ver tabela abaixo). Mas em vez
de qualquer uma dessas opções ou um sinônimo estes, as palavras "so"
e "e" foram aplicadas em seu lugar, fazendo parecer que a noite
anterior à luz do dia.
Strong
#
King
James
Version
da
Bíblia
Hebraico Strong Definições Brown-driver-
Briggs Hebrew
Lexicon
Novo
Significado
Comprehensive
1961 e hayah de existir, ou tornar-se, vir a
passar, farol,
completamente;seguir,aconteceu;
ser, para se
tornar,
aacontecer, para
existir, a
acontecer, para
cair;
em seguida,
veio
ou
seguido
Aqui está como o texto deve ser:
Dirigindo-lo pelo nome, Elohiym chamou à luz dia, ao contrário de
noite, e às trevas chamou noite, ao contrário do dia. Depois veio o
anoitecer (crepúsculo), depois veio o amanhecer (crepúsculo), um dia.
Êxodo 01:05
Pergunta 15: Agora, com uma compreensão mais completa das
diferenças do dia e da noite, como declarado pelo Criador, durante o
período de tempo que Ele fez todas as Suas obras da criação?
E a terra produziu erva, a erva que dá semente segundo a sua espécie,
ea árvore que dá fruto, cuja semente está nela conforme a sua espécie.
E [Elohiym] viu que era bom. Depois veio o anoitecer (crepúsculo),
depois veio o amanhecer (crepúsculo), dia três. Gênesis 1:12-13
Resposta: O foco da criação foi a beleza do que é visível na luz, então
parece que todas as obras criativas foram realizadas durante o tempo
da luz maravilhosa de as horas de luz do nascer ao pôr do sol.
Absolutamente nada foi criado durante a escuridão da noite, após o
Criador provocou a "luz".
Aqui abaixo é um gráfico simples delinear círculo diário da criação do
tempo, como é descrito na Bíblia
Criação do Diário Círculo do Tempo
DIA
ONE
1. Sol a sol, em que foram feitas as obras criativas -
Yahuah convocado luz ea divisão do dia e da
noite foram estabelecidas.
2. Depois veio o anoitecer (crepúsculo),
3. Depois veio o amanhecer (crepúsculo), um dia.
SEGUNDO
DIA
1. Sol a sol, em que foram feitas as obras criativas -
Yahuah dividiu as águas acima e abaixo do
firmamento.
2. Depois veio o anoitecer (crepúsculo),
3. Depois veio o amanhecer (crepúsculo), dia dois.
TERCEIRO
DIA
1. Sol a sol, em que foram feitas as obras criativas -
Yahuah trouxe terra seca, grama, ervas e
sementes de rolamento.
2. Depois veio o anoitecer (crepúsculo),
3. Depois veio o amanhecer (crepúsculo), dia três,
etc, etc
Pergunta 16: Em Gênesis 1, Moisés registra o Criador declarando seis
vezes acima do círculo de tempo, um dia, dois, três, etc Então, quando
o sétimo dia é mencionado no Capítulo 2, o período de tempo que o
Criador designar como Santo? Era um círculo completo de tempo, a
luz do dia, seguido pela noite, ou foi apenas para ser a luz do dia?
E no sétimo dia [Elohiym] terminou a obra que tinha feito, descansou
no sétimo dia de toda a Sua obra, que tinha feito. 3 Então [Elohiym]
abençoou o sétimo dia eo santificou, porque nele descansou de toda a
obra que [Elohiym] criara e fizera. Gênesis 2:2-3 NVI
Resposta: As horas sagradas do sábado são apenas a parte à luz do dia
o círculo completo de 24 horas do tempo, e são contados por horas. Em
nenhum lugar nas Escrituras há uma pitada de vigílias da noite que
ocorrem no sábado. Se o dia estava para começar ao pôr do sol, este
versículo da Bíblia seria o lugar onde seria abertamente declarado pelo
Criador, afirmando que ele descansou no sétimo noite, três vezes, em
vez do sétimo dia. Veja João 11:09. A verdade fundamental
estabelecido em Gênesis 01:05 foi a divisão permanente da luz do dia
da noite.
Toda a criação foi feito durante a luz do dia, por isso é lógico que,
quando o Criador cessado ou descansou no sétimo dia, foi durante o
mesmo período de tempo que Ele anteriormente fez seu trabalho de
criar.
Pergunta 17: Que exemplos das Escrituras mostram que o Pai Celestial
projetou as realidades físicas da luz e das trevas com uma aplicação
simbólica espiritual?
Resposta: Em toda a Escritura Yahuah usou esses princípios
contrastantes de luz e escuridão para ensinar suas verdades espirituais
Trevas era representar metaforicamente infidelidade, cegueira
espiritual e do mal, enquanto a luz era representar a verdade, a
compreensão espiritual e justiça.. Yahushua usa esses princípios mais e
mais ao longo de seu ministério junto com Paul.Mas a vereda dos
justos é como a luz da aurora, que vai brilhando mais e mais até ser
dia perfeito. O caminho dos ímpios é como a escuridão: não sabem eles
em que tropeçam. Provérbios 4:18-19
Yahushua, o Messias disse: "Eu sou a luz do mundo: quem me segue
não andará em trevas, mas terá a luz da vida." João 8:12
"Vós sois a luz do mundo". Mateus 5:14
"Deixe sua luz brilhe diante dos homens" Mateus 5:16
Nele estava a vida, ea vida era a luz dos homens. E a luz resplandece
nas trevas e as trevas não a compreenderam. João 1:4
Este veio como testemunha, para dar testemunho da luz que todos os
homens cressem por meio dele. João 1:7
"Porque todo aquele que faz o mal odeia a luz, e não vem para a luz,
para que as suas obras não sejam reprovadas. Mas aquele que faz, vem
para a luz, para que as suas obras sejam manifestas, porque são feitas
em [Elohiym] . " João 3:20
Mas vós sois a geração eleita, o sacerdócio real, a nação santa, o povo
adquirido, para que vos anunciar as virtudes daquele que tinha
chamou das trevas para a sua maravilhosa luz. 1Pedro 2:09
Mas todas estas coisas, sendo condenadas, se manifestam pela luz, pois
tudo o que se manifesta é luz. Efésios 5:13
Pergunta 18: O que Yahuah, nosso Criador nos dá para determinar os
nossos dias e os nossos anos, luzes ou escuridão?
Então [Elohiym] disse: "Haja luzeiros no firmamento dos céus para
separar o dia da noite, e sejam eles para sinais e para estações, e para
dias e anos Gênesis 1:14 NTLH.
Toda boa dádiva e todo dom perfeito são lá do alto, descendo do Pai
das luzes, em quem não pode existir variação ou sombra de mudança.
Tiago 1:17 NTLH
Resposta: Luzes! Observe que não diz noites e as trevas, mas as luzes
foram dadas para separar o dia da noite. O "dia" é regido pelos raios
diretos do sol, ocupando o tempo do nascer ao pôr do sol. As luzes da
noite ligar quando o pôr do sol, como uma a uma as estrelas começam
a brilhar.
Além disso, o nosso Criador é referido como o "Pai das Luzes", em que
não há sombra de uma órbita de viragem. O sol e as estrelas nunca tem
sombras, então esta referência de "sem sombra" é só para a lua.
Quando a lua representa o "Pai das Luzes" é somente quando não há
sombra de sua órbita de viragem. Há apenas uma fase lunar que se
encaixa nessa descrição, e que é a "cheia de" Lua Nova. Louve
Yahuah!
Pergunta 19: Se um "dia" começa ao pôr do sol, quando se torna
escuro, então quando é que uma "noite" começa?
Quando os gazeu foi dito: "Sansão chegou aqui!" eles cercaram o local
e ficaram à espera dele a noite na porta da cidade. Eles ficaram quietos
a noite toda, dizendo: "Na parte da manhã, quando é a luz do dia,
vamos matá-lo." Juízes 16:02 NVI
Fazes as trevas, e vem a noite, na qual todos os animais da floresta
sobre fluência. Salmos 104:20 NKJV
Enquanto a terra durar,
sementeira e colheita,
frio e calor,
verão e inverno,
e de dia e de noite
não cessarão.
Gênesis 8:22 KJV
Resposta: Dia e noite não tanto iniciar simultaneamente no mesmo
momento. Se assim for, que seria a mesma coisa. Um dia começa
quando ela se torna a luz do dia (raios diretos do sol) de acordo com
Juízes 16:02. Noite começa quando se torna escuro, o tempo em que os
animais começam a rastejar sobre o acordo com o Salmo 104:20.
O mais impressionante dos exemplos acima é a terceira em Gênesis
8:22, que ilustra esse ponto com quatro comparações únicas. Cada
uma destas comparações são opostos totais encontrados na natureza,
ilustrando o dia ea noite como o epítome dos opostos.
Se isso não bastasse, observe que cada um dos exemplos de
comparação são dadas em sua ordem precisa: sementeira deve
anteceder a colheita, em todos os casos. Existia frio antes da criação do
sol. Porque o ano bíblico começa na primavera, o verão vai ocorrer
pela primeira vez como o ano termina com o inverno. Da mesma
forma, o dia começa antes da noite em perfeita harmonia com os três
exemplos anteriores. Cada uma destas comparações são pontuadas
com o ponto dado a Noé ao sair da arca, que estes devem permanecer
perpetuamente na terra e não cessarão.
Pergunta 20: É "meia-noite" no meio da noite ou no meio do dia?
E assim foi, ao meio-dia, Elias zombava deles e disse: "Clama em alta
voz, pois ele é um [Elohiym]; pode ser que esteja meditando, ou ele está
ocupado, ou ele está em uma viagem, ou talvez ele está dormindo e
deve ser despertado. " 1 Reis 18:27 NVI
Resposta: É o meio da "noite", porque a meio do dia vem depois da
manhã e antes do por do sol e é chamado de meio-dia.
Pergunta 21: Quando as Sagradas Escrituras falam sobre o romper do
dia em 2 Samuel 2:32, Juízes 19:25 e Jó 07:04; isso é falar de nascer ou
pôr do sol?
Então levantaram a Asael eo sepultaram no túmulo de seu pai, que
estava em Belém. E Joabe e seus homens caminharam toda aquela
noite, e eles vieram até Hebrom ao amanhecer. 2 Samuel 2:32 NTLH
Mas os homens não acatam ele. Então, o homem pegou da sua
concubina e levou-a para fora para eles. E eles sabiam que ela e abusou
dela durante toda a noite até de manhã, e quando o dia começava a
quebrar, eles deixá-la ir. Juízes 19:25 NVI
Resposta: Ambos estão falando do nascer do sol. Nós podemos saber
com certeza que a alvorada se fala aqui é específico ao nascer do sol,
porque é claro que essas coisas ocorreram durante toda a noite. O
período de crepúsculo é conhecida como a quarta vigília da noite do
Daybreak Watch. Próprio Daybreak é sinônimo de o sol nascer.
Pergunta 22: Foi o sol feito para governar o dia ou foi feito para
governar a noite?
Então [Elohiym] disse: "Haja luzeiros no firmamento dos céus para
separar o dia da noite, e sejam eles para sinais e para estações, e para
dias e anos, e sejam eles para luminares na expansão dos os céus, para
alumiar a terra ", e assim foi. Então [Elohiym] fez duas grandes luzes:
a maior para governar o dia, eo luminar menor para governar a noite.
Ele fez também as estrelas. [Elohiym] colocou-os no firmamento dos
céus, para alumiar a terra, e para governar o dia ea noite, e para
separar a luz das trevas. E [Elohiym] viu que era bom. Gênesis 1:14-18
NVI
Resposta :: Foi feito para governar o dia e não a escuridão da noite.
Pergunta 23: Uma vez que o sol foi feito para governar o dia, então
como pode um dia começa à noite, quando o sol não está governando
ou medir?
Àquele que fez os grandes luminares, porque a sua misericórdia dura
para sempre, o sol para governar de dia, porque a sua misericórdia
dura para sempre, a lua e as estrelas para presidirem a noite, porque a
sua misericórdia dura para sempre. Salmos 136:7-9 NVI
Resposta: Não pode, isso é impossível porque o Criador projetou esses
três tipos de luzes para declarar as divisões específicas de cada data do
calendário lunar. Dia e noite simplesmente nunca podem se sobrepor
mais do que plena luz ea escuridão completa pode sobrepor-se, como
as trevas desaparecem, produzindo apenas plena luz. A escuridão é
apenas a ausência de plena luz, assim, portanto, "noite" é a ausência
de "dia".
Pergunta 24: É declarado várias vezes ao longo das Escrituras, que o
Criador dividiu o dia ea noite, ou a luz das trevas (Gênesis 1:3-5, 14-
19). Então, se o dia começa ao pôr do sol, isso levanta a questão,
quando foi Ele juntá-los novamente?
Nunca. Nada pode ser adicionado a ela, e nada feita a partir dele.
[Elohiym] faz isso, que os homens temam diante dele. Eclesiastes 3:14
NTLH
A erva seca, a flor murcha, mas a palavra de nosso [Elohiym]
permanece para sempre. Isaías 40:8 NTLH
Resposta: Ele nunca colocá-los juntos novamente. Muitos optam por
acreditar que a luz do dia segue a noite nas Escrituras 24 horas data
calendário lunar. Isso é porque eles acreditam que o dia começa ao pôr
do sol, mas essa crença não é baseada em peso da Escritura de provas.
Pergunta 25: É possível para os meus pais que visitam a permanecer
em casa o dia todo, sem passar a noite?
Fazes as trevas, e vem a noite, na qual todos os animais da floresta
sobre fluência. Salmos 104:20 NKJV
Resposta: Sim, porque quando um dia termina, a noite começa.
Pergunta 26: Em Números 11:32, diz que os filhos de Israel se
reuniram a codorna todo aquele dia e toda aquela noite, e todo o dia
seguinte. Então, quantos dias e quantas noites eles se reúnem codorna?
E o povo ficou acordado que todos os dias, durante toda a noite e todo
o dia seguinte, e colheram as codornizes (aquele que colheu menos,
colheu dez home runs), e as estenderam para si ao redor do arraial.
Números 11:32 NVI
Resposta: Eles se reuniram codorna por dois dias e uma noite, mas se
um dia começa à noite, em seguida, eles teriam se reuniram codorna
três dias e três noites. Mas a evidência mais profunda no versículo
acima é a referência que a noite pertenceu ao primeiro dia
mencionado, com a referência do segundo dia mencionado como o "dia
seguinte".
Pergunta 27: Se o dia começa ao nascer do sol, então por que os
versículos bíblicos que descrevem quando o Salvador ressuscitou, diga
"de manhã cedo", enquanto ainda estava escuro?
Resposta: Ótima pergunta. Um erro foi descoberto nas traduções que
colocam a palavra "cedo" ou "início da manhã" no lugar da palavra
grega "proi" 4404 #. Embora tenha sido regularmente traduzido para
significar "no início da manhã", que na verdade significa que durante
a "quarta vigília da noite" ou "Daybreak Watch" ou Ver Marcos 16:2,
9 "de madrugada.", João 20:01; Mateus 20:01
Concordância de Strong Definição grego: # 4404 (proi) = ao
amanhecer; por implicação, o DaybreakWatch. 1. Proi é a quarta
vigília da noite.
Por favor, não perca este ponto. "Amanhecer e entardecer" são ambos
equacionada com a "noite" e não com o "dia". É nestes versículos
bíblicos muito destinados a provar que o amanhecer faz parte do dia
em que o peso da evidência é dada pelo contrário. Dawn é sinônimo
com a última vigília da noite. Estes versos incluem todas as referências
de tempo no Novo Testamento para a Ressurreição. O texto grego
esclarece que a ressurreição realmente aconteceu no "primeiro" da
semana, que foi também a data do calendário lunar 16. Mas o que é
mais surpreendente, é que a ressurreição ocorreu no final da data do
calendário lunar do dia 16, durante a quarta vigília da noite. Devido a
esta descoberta de como data do calendário lunar do Criador é
dividido, agora é possível contabilizar o Messias estar no túmulo por
três dias e três noites. Em breve - Consulte o artigo na íntegra, "três
dias e três noites, o sinal profético de Jonas."
Pergunta 28: Se Yahuah deu a celeste maná somente com a finalidade
de alimentar o povo e ilustrando que dia foi o sétimo dia de sábado ,
então por que há tanta escrito em Êxodo 16 sobre outras unidades de
tempo definidos pelo maná diariamente?
Resposta: Em ternos cuidados do Pai Celestial, Ele fez chover maná do
céu durante quarenta anos no deserto para alimentar os israelitas
recém-lançado quem levaram para fora do Egito. Além de esclarecer
que dia foi o sábado do sétimo dia, existem vários outros detalhes
centric tempo relacionadas com o maná de grande importância para
Yahuah, que ilustram quando o dia começa.
Ele deu especificações precisas sobre a reunião do maná como Ele
afirmou que seria usado para testar a sua obediência a ele. Era Seu
plano ordenado de usar uma programação de alimentação para
restaurar Israel de volta para Seu sistema tempo lunissolar.
. . . Que eu possa prová-los, se anda em minha lei ou não. Êxodo 16:04
. . . as pessoas devem sair e reunir o dever de cada dia. . . Êxodo 16:04
. . . no sexto dia, que prepararão o que colherem; e será o dobro do que
colhem cada dia. Exodus16: 5
. . . Pela manhã, vereis a glória de [Yahuah]. . . Êxodo 16:07
pela manhã vos fartareis de pão, e sabereis que eu sou [Yahuah] seu
[Elohiym]. Êxodo 16:12
E, quando o orvalho que estava se levantou, eis que sobre a face do
deserto estava uma coisa miúda, redonda, miúda como a geada sobre a
terra. Êxodo 16:14
Ninguém deixe dele para amanhã. Exodus16: 19
. . . alguns deles deixaram dele para a manhã [próximo], e criou bichos,
e cheirava mal. . . Êxodo 16:20
. . . quando o sol se acendeu, derretia-se. Êxodo 16:21
. . . no sexto dia eles se reuniram duas vezes mais. . . Êxodo 16:22
Amanhã é repouso, sábado santo [Yahuah]: coza o que vos vai assar
hoje, e faze que vos vai ferver, eo que permanece sobre ajuntai para
que você possa ser mantido até a manhã. Êxodo 16:23
E guardaram-no até o dia seguinte, como Moisés tinha ordenado, e não
cheirou mal, nem houve nele bicho algum. Êxodo 16:24
E disse Moisés: Comei-o hoje, porquanto hoje é o sábado para o
[Yahuah]:-a-dia não o achareis no campo. Êxodo 16:25
Seis dias o colhereis, mas o sétimo dia, que é o sábado, nele não haverá.
Êxodo 16:26
Veja, por que [Yahuah] vos deu o sábado, por isso ele lhe dá no sexto
dia o pão para dois dias; cada um fique no seu lugar, não saia ninguém
do seu lugar no sétimo dia. Êxodo 16:29
Assim repousou o povo no sétimo dia. Êxodo 16:30
Aqui verifica-se que o sábado do Pai Celestial é o sétimo dia e que
segue seis dias de trabalho. Verificou-se que começa o dia de manhã,
em vez de durante a noite. O maná poderia ser preservado para
exatamente 24 horas a partir do momento em que foi dado, e sempre
foi ruim na manhã seguinte, quando o fresco, novo maná foi fornecido,
exceto no sábado. Nós descobrimos que o termo hoje é referido como
um dia, que se inicia no dia. No sexto dia, Moisés falou de amanhã
como sendo o dia de sábado. Se o sábado começou ao pôr do sol,
Moisés teria falado a respeito da noite de sábado ou ao pôr do sol. O
sábado começou na parte da manhã, como descrito por nenhum maná
dado naquela manhã. Em todo este Êxodo 16 narrativa não há uma
menção da palavra noite, pôr do sol, crepúsculo ou à noite. O peso da
evidência declara nesta história só que o dia começa na manhã
seguinte.
Pergunta 29: Desde o tempo de Moisés, até o morte do Salvador na
cruz, um período de 1500 anos, tanto pela manhã e sacrifícios noite
foram uma ocorrência diária. Era só o sacrifício da manhã que foi
administrada durante o dia, com os sacrifícios da noite sendo
administrado durante a escuridão da noite?
Resposta: Não! Ambos os sacrifícios foram administradas durante o
dia. O sacrifício da manhã foi administrada logo após o nascer do sol,
enquanto o sacrifício da tarde foi administrada pouco antes do por do
sol. De acordo com escrituras, a noite foi igualado com a troca de luz
que ocorreu ao amanhecer e ao pôr do sol novamente. Este é um
conceito novo para a maioria, mas traz todos os pontos nas Escrituras
em harmonia, sobre este assunto de quando um dia começa. (Êxodo
29:39).
Pergunta 30: De acordo com Exodus18: 13, quando é que "no dia
seguinte" começar, à noite, a noite ou de manhã?
E assim foi, no dia seguinte, Moisés assentou-se para julgar o povo, eo
povo estava em pé diante de Moisés desde a manhã até a noite.
Exodus18: 13 NVI
Resposta: O "dia seguinte", começa na parte da manhã, como todos
das Escrituras indicam claramente. Você nunca vai encontrar a
Escritura declara que o "dia seguinte", ou amanhã começa à noite ou a
noite, mas sempre começa ao nascer do sol.
Pergunta 31: De acordo com Êxodo 16:22-25, quando é que o resto do
sábado santo Yahuah começar, no meio da noite, à noite ou de manhã?
E assim foi, no sexto dia, que eles se reuniram pão em dobro, dois
omers para cada um. E todos os príncipes da congregação vieram, e
contaram Moisés. Então ele lhes disse: "Isto é o que [Yahuah] disse:
'Amanhã é um descanso sabático, sábado santo ao [Yahuah] Asse o
que você vai assar hoje, e ferva o que você vai ferver, e ajuntai para
vós. tudo o que resta, para ser guardado até a manhã ". "Então eles
guardaram-no até de manhã, como Moisés tinha ordenado, e não
cheirou mal, nem houve quaisquer vermes nele. Então Moisés disse:
"Comei-o hoje, porquanto hoje é o sábado [Yahuah], hoje você não vai
encontrá-lo no campo Êxodo 16:22-25 NTLH.
Resposta: Observe que a referência feita por [Yahuah] foi feito no dia
da preparação, declarando que amanhã é o descanso sabático. O
sábado começa na parte da manhã como todos os outros dias começar.
Então, quando naquela manhã seguinte chegou, temos uma declaração
de confirmação de Moisés que disse: "Comei-o hoje, porquanto hoje é
o sábado [Yahuah]." Isso deixa claro que o sábado começou no dia
seguinte ao amanhecer.
Pergunta 32: Se uma noite é considerado um dia, então o que Yahuah
dizer quando disse que iria chover sobre a terra quarenta dias e
quarenta noites?
Para depois de mais de sete dias farei chover sobre a terra quarenta
dias e quarenta noites, e eu o destruirei de sobre a face da terra todos
os seres vivos que eu fiz. Gênesis 07:04 NVI
Resposta: A noite não é considerado o dia, ou até mesmo o início de um
dia, mas, em vez dias e noites são opostos exactos. Além disso, o aviso
prévio, nas palavras de Yahuah Si mesmo, Ele afirma que o "dia" vem
antes do "noite", quando disse que choveria quarenta dias e quarenta
noites. O Pai Celestial sempre faz declarações precisas, e Ele não disse
que chover-
Pergunta 33: Por que é que os homens justos são despertou contra os
hipócritas? O que as pessoas fazem para merecer ser chamado de
hipócritas?
Os retos estão surpresos com isso, e os inocentes agita-se contra o
hipócrita. No entanto, o justo irá realizar para o seu caminho, e ele que
tem as mãos limpas será mais forte e mais forte. "Mas, por favor, volte
mais uma vez, a todos vocês, pois eu não encontrar um homem sábio
entre vós, meus dias passaram, meus propósitos são quebrados, mesmo
os pensamentos do meu coração Eles mudam a noite em dia;. '. A luz
está perto ', dizem eles, em face da escuridão. Job 17:8-14 NVI
Resposta :: Porque os hipócritas estão dizendo que Yahuah, nosso
Criador, é um mentiroso quando ele disse que separou o dia da noite.
Gênesis 1:04. Agora eles mudaram a noite em dia, quando Ele havia
declarado a luz para ser o dia e não a escuridão da noite.
Pergunta 34: Se o dia foi destinado desde a criação até início da noite,
então como alguém poderia mudar a noite em dia, tal como indicado
neste versículo, se ele já foi?
E a palavra do [Yahuah] veio a Jeremias, dizendo: "Assim diz
[Yahuah]:" Se você pode quebrar a minha aliança com o dia eo meu
pacto com a noite, de modo que não haverá dia e noite a seu tempo, em
seguida, a minha aliança também pode ser quebrado com Davi, meu
servo, de modo que ele não terá um filho que reine no seu trono, e com
os levitas, sacerdotes, meus ministros. Jeremias 33:19-21 NTLH
Resposta: Isso seria totalmente impossível. Pelos maus israelitas que
tentam fazê-lo, provocaria a destruição total da casa de Israel.
Pergunta 35: De acordo com as palavras do Altíssimo, que é a
diferença entre um dia e uma noite?
Assim diz [Yahuah], que dá o sol para luz do dia, as ordenanças da lua
e das estrelas para luz da noite, que agita o mar e suas ondas rujam
([Yahuah] dos Exércitos é o seu nome): "Se estas ordenanças de diante
de mim, diz [Yahuah], então a semente de Israel deixam também de
ser uma nação diante de mim para sempre." Jeremias 31:35-36 NTLH
Àquele que fez os grandes luminares, porque a sua misericórdia dura
para sempre - o sol para governar de dia, porque a sua misericórdia
dura para sempre, a lua e as estrelas para presidirem a noite, porque a
sua misericórdia dura para sempre. Salmos 136:7-9 NVI
Resposta: Day é igual a luz solar, e de noite é igual ao período com a
lua e as estrelas. Jeremias 31:35 e Salmo 136:7-9. Gênesis 1:05 diz:
"[Elohiym] chamou à luz dia, e às trevas chamou noite.
Pergunta 36: Em Jeremias 33:19-21, Yahuah fala sobre o mal que Ele
traria sobre a casa de Davi e do reino de Israel se eles quebraram seus
convênios. Qual foi o pacto que Ele foi sublinhando que o seu povo não
quebrar?
Assim diz [Yahuah], que dá o sol para luz do dia, as ordenanças da lua
e das estrelas para luz da noite, que agita o mar e suas ondas rujam
([Yahuah] dos Exércitos é o seu nome): "Se estas ordenanças de diante
de mim, diz [Yahuah], então a semente de Israel deixam também de
ser uma nação diante de mim para sempre." Jeremias 31:35-36 NTLH
Resposta: Isso está se referindo especificamente aos Seus preceitos do
"sol" para luz do dia, e as ordenanças da "lua e as estrelas" para uma
luz da noite. Isto é revelar Sua pedaço de tempo específico ordenado
lunar-solar celestial. Se essas divisões ordenados de tempo são
quebradas ou modificadas que causaria a 12 horas "dia", e os quatro
relógios da "noite", não estar em suas posições de tempo divinamente
designado.
Este versículo tem uma maior aplicação do que aparece pela primeira
vez na superfície. Ele está afirmando que, se as ordenanças de "sol"
para dias e "lua e as estrelas" para as noites sai, assim será a "semente
de Israel" cessar de ser uma nação para sempre. "Em outras palavras,
este é um confirmado prometida de Yahuah que Ele nunca vai mudar
seu sistema de tempo astro-luni-solar por Deus. Isso é garantido no
fato de que, "se você é [o Messias], também sois descendentes de
Abraão, e herdeiros conforme a promessa." Gálatas 3 :28-29. O ponto
aqui é que haverá uma segunda vinda do Messias para os remidos do
Terra. Todos estes remidos do terra, desde o tempo de Abraão, são ou
judeu ou gentio, mas se ser o Messias de todos eles são Israel espiritual,
a semente de Abraão e será recompensado com a vida eterna.
Portanto, a garantia é que a semente fiel de Abraão nunca cessará,
assim como as ordenanças do sol, a lua e as estrelas que representam
as divisões do "dia "e" noite "deve nunca cessar.
Por outro lado, logo depois que os planos da crucificação foram
estabelecidas pelos líderes judeus para mudar calendation de Israel. A
Lua Nova foi alterado a partir da "lua cheia" para o "primeiro
crescente visível." O ciclo semanal romano foi adotado para o sábado
do sétimo dia, eo "dia" foi alterado para começar ao pôr do sol, em vez
de nascer do sol. Vai ser todos aqueles que continuam a andar no
tempo artificial dos judeus e romanos, que deixará de ser uma nação
para sempre, de acordo com o versículo acima. Daniel 7:25 esclarece
que os tempos e as leis de Yahuah seriam adulterados pelo homem.
Pergunta 37: O que [Yahuah] declarar que acontecerá se o decreto-lei
de dia e à ordenação de noite são removidos?
Assim diz [Yahuah], que dá o sol para luz do dia, as ordenanças da lua
e das estrelas para luz da noite, que agita o mar e suas ondas rujam
([Yahuah] dos Exércitos é o seu nome): "Se estas ordenanças de diante
de mim, diz [Yahuah], então a semente de Israel deixam também de
ser uma nação diante de mim para sempre." Jeremias 31:35-36 NTLH
Resposta: Israel deixará de ser uma nação. O foco é em Israel pela
simples razão de que tinha sido Israel que teve a honra de manter o
sistema de tempo correto Yahuah como um farol de verdade sempre
antes de as nações da terra. Poderia ser que a reforma cronometragem
surgiu após a crucificação do Messias, como os líderes perceberam o
serviço do Templo e seu modelo de calendário astro-luni-solar
continuamente lembrou-os de sua culpa. Pois foi tanto os serviços do
Templo e do calendário que prefigurou toda a obra do Messias. Esta
mudança na calendation, por líderes de Israel e Roma foi profetizado
para ocorrer por Daniel o Profeta (Daniel 7:25).
Pergunta 38: Será que não Levítico 23:32 suporte o "dia" do início ao
pôr do sol e termina ao pôr do sol, como ele afirma, "do mesmo até
mesmo você deve celebrar seus sábados?"
Resposta: Muitas sabatistas têm usado Levítico 23:32 como uma
segunda testemunha de Gênesis 1:5, declarando que devemos santificar
o sábado do sétimo dia de "até a mesmo." Eles interpretam "do mesmo
até mesmo", como "a partir de pôr do sol até pôr do sol. " Enquanto
isto parece provável, à primeira vista, a verdade pura e não adulterada
está dentro do texto original hebraico e não é encontrada nas
modernas versões traduzidas como ilustrado abaixo.
Ele vos será o sábado de descanso solene. . . desde a tarde até a noite,
você deve celebrar o seu sábado. "Levítico 23:32 NVI
Este versículo está declarando os parâmetros da festa "Dia da
Expiação", que é anual "dia de festa" que ocorrem a cada ano. É,
simultaneamente, fixado ao "décimo data" da contagem mês a partir
da "cheia" dia de Lua Nova, e para o segundo dia da semana lunar.
Isso é verificado no fato de que Festa dos Tabernáculos é no dia 15 do
mês, e consistentemente fixado ao sábado do sétimo dia.
Este versículo em Levítico 23:32 não está se referindo especificamente
ao sábado do sétimo dia não mais do que a descrição da Festa da
Páscoa no versículo 5. No entanto, parece que os parâmetros de tempo
para esta festa, "Dia da Expiação", que também se aplicam ao sábado
do sétimo dia.
De acordo com a Escritura amanhecer e entardecer são equiparados
com o primeiro eo quarto vigílias da noite. O crepúsculo amanhecer é
chamado de manhã ou Daybreak Watch, enquanto o crepúsculo
anoitecer é chamado de Noite Watch.
A palavra que é traduzida em Levítico 23:32, para ser "mesmo", é a
mesma palavra que é traduzida em verso Levítico 23:05 como
"Crepúsculo". A palavra hebraica pode ser derivado tanto # 6153
`Ereb ou # 6154 como` Ereb, pois estas são as mesmas palavras
hebraicas exatas com definições diferentes.
Strong # King
James
Version da
Bíblia
Hebraico Strong Definições Brown-driver-Briggs
Hebrew Lexicon
6153 a noite `Ereb
`Ereb #
6154 -
Palavra -
Root
# 6148
`árabe
Palavra
Raiz -
# 6150
`árabe
anoitecer; maré da
noite, noite, dia
mistura
trançar, intermix;
se misturam; dar
penhores;
envolver, dar para
ser de
segurança (como
uma espécie
detroca);
através da idéia de
cobrir com textura;
sol; ser escurecido;
pôr do sol, noite,noite;
# 6154 `Ereb - que é a
mesma palavra hebraica:
trama,
como misto;entrelaçados;
material de malha;
mistura; pessoas mistos;
sociedade de economia
mista.
a troca, para envolver,
dar em penhor, para
assumir um
compromisso, para
garantir;
para troca de promessas;
ter comunhão com;
O segredo para desvendar este mistério é em hebraico a palavra # 6154
`Ereb, o que significa mistura de luz ou crepúsculo.
Dicionário Definição de Webster:
crepúsculo = 1: a luz do céu entre a noite eo nascer do sol pleno ou
entre o pôr e noite completo, produzido pela difusão da luz solar
através da atmosfera e sua poeira.
Curiosamente, não há apenas um, mas dois períodos de crepúsculo
conectado com cada data do calendário 24 horas. Em primeiro lugar,
há o crepúsculo que precede nascer do sol, bem como o crepúsculo que
segue pôr do sol. Mas, mais específico ainda, é nesse período culmina
em uma troca de luz absoluta. Estes incluem o período das trevas para
a completa troca radiante luz do sol ao nascer do sol, ou a luz do sol
radiante para troca escuridão ao pôr do sol (à noite).
Já foi estabelecido que o Criador está interessado apenas em porções
de luz do dia como os raios radiantes de luz solar simbolizam verdade e
da justiça. Portanto, a frase "desde a tarde até mesmo está realmente
se referindo ao crepúsculo anterior e seguindo as horas do dia,
comportando-se como suportes para a parte sagrada deste dia de festa
da Expiação.
Pensar fora da caixa, é possível, em qualquer nível que a frase "desde a
tarde até mesmo" foi destinado por Moisés para significar "a partir de
crepúsculo até o crepúsculo", as trocas de luz que ocorrem tanto o
nascer eo pôr do sol? Isto parece mais provável.
Com isto em mente, vamos voltar a Levítico 23:32, é de todo possível
que Moisés é esclarecer "de crepúsculo até o crepúsculo, você deve
comemorar seu sábado na data lunar décimo? Poderia estar se
referindo ao nascer e pôr do sol troca de luz ? Este é um período de 12
horas de tempo e não pode incluir um 24 horas data calendário lunar
cheia de uma revolução solar completa. Como resultado "do mesmo
até mesmo," seria sinônimo de "de sol a sol", uma mera 12 horas
período.
Se isso for verdade, então "até mesmo (nascer do sol) até mesmo (por
do sol)" só seria a parcela de tempo entre o ponto A eo ponto B. Estes
estão definindo os parâmetros de apenas luz do dia, começando ao
nascer do sol e termina ao pôr do sol. Uma vez que esta porção de
tempo definido é um período entre dois "trocas de luz," só há duas
opções a respeito de quando esse espaço de tempo é. Teoricamente,
poderia ser do pôr ao nascer do sol, que engloba apenas o escuro da
noite, ou de sol a sol abrangendo apenas a luz do dia. Mas, nós temos
isso na boa autoridade de Moisés e Yahuah que esta parcela 12 horas
de uma data do calendário lunar era de fato do nascer ao pôr do sol,
pois o simples fato de que esta era a terminologia usada para descrever
a "luz do dia", como é chamado " Dia "da Expiação e não a" Noite
"da Expiação. Consulte o estudo da palavra, Gênesis 1:1-5 "Quando
um dia começa."
Pergunta 39: Gênesis, Êxodo, Levítico e foram os três primeiros livros
escritos por Moisés. Se os elementos de tempo esclarecidas em cada um
desses livros não estavam em perfeita harmonia uns com os outros,
Moisés teria questionado Yahuah. Foi Moisés quem gravou os Dez
Mandamentos, inclusive o quarto, que fala especificamente do sábado
do sétimo dia. Existe alguma referência na Bíblia para iniciar o sábado
à noite?
Resposta: Na Bíblia, não há nenhuma referência ao sábado início da
noite ou do sol, além dos erros de tradutor de Gênesis 1 e Levítico
23:32, como mencionei anteriormente. O quarto mandamento diz
claramente que o Sabbath era ocupar o período de tempo conhecido
como a luz do dia e qualquer referência é feita para a escuridão da
noite.
Lembre-se do dia de sábado, para santificá-lo. Seis dias trabalharás, e
farás toda a tua obra, mas o sétimo dia é o sábado [Yahuah] seu
[Elohiym]. Nele você deve fazer nenhum trabalho: você, nem o teu
filho, nem tua filha, nem o teu servo, nem a tua serva, nem o teu
animal, nem o estrangeiro que está dentro das tuas portas. Porque em
seis dias [Yahuah] fez os céus ea terra, o mar e tudo o que neles há, e
ao sétimo dia descansou. Portanto [Yahuah] abençoou o dia do sábado,
eo santificou. Êxodo 20:8-11 NVI
No entanto, alguns gostariam de nos fazer pensar que, após a
crucificação de Yahushua as pernas dos dois criminosos foram
quebradas e que, juntamente com o Messias, foram removidas de suas
cruzes, porque o "sétimo dia" foi com base na em três horas. No
entanto, nós sabemos para um fato que a Páscoa era para começar em
três horas como era para começar ao pôr do sol. O sábado imediato
que estava chegando na festa da Páscoa era sábado, o que era para
começar precisamente ao pôr do sol. Todos os homens judeus juntos
com suas famílias, eram para estar em casa para o evento e foi-lhes
ordenado "para não pisar fora da porta de suas casas até a manhã"
(Êxodo 12:22). O sábado do sétimo dia era para começar na manhã
seguinte ao nascer do sol, no momento exato da Páscoa terminou. O
sábado do sétimo dia nunca é mencionado nas Escrituras como a
sobreposição, mas era sempre o dia seguinte da Páscoa e começou ao
amanhecer. Era a festa de sete dias dos pães ázimos, que sempre
começou no sábado do sétimo dia, tornando-se uma alta sábado.
Mark 15:42-44 Agora, quando já era tarde, porque era a preparação,
isto é, a véspera do sábado, José de Arimatéia, um membro do
conselho de destaque, que também esperava o reino de [Elohiym], indo
e tomando coragem, foi ter com Pilatos e pediu o corpo de Jesus.
A referência a "quando já era tarde" é claramente indicado para ser
um período de tempo antes do início do sábado. Era a continuação da
preparação, mas foi o início da Páscoa que já havia começado ao pôr
do sol. . . A palavra "dia" não existe no texto original. A palavra # 4315
prosabbaton simplesmente significa antes do sábado. O outro
identificador neste versículo declara que o sol já se pôs, como este é o
significado da palavra # 3798 opsias, o que significa o cair da noite.
Observe como esse novo testamento a palavra # 3798 noite de outono
está em harmonia com o Antigo Testamento # 6153 Ereb que significa
crepúsculo. É evidente que o sábado ainda estava para começar na
manhã seguinte junto com a Festa dos Pães Ázimos. Era a festa da
Páscoa, que tinha acabado de começar ao pôr do sol no dia 14.
Agora era o dia da preparação da Páscoa, e cerca da hora sexta. E
disse aos judeus. . . João 19:14
Agora, no lugar onde Ele foi crucificado havia um jardim, e no jardim
um sepulcro novo, em que ninguém ainda havia sido posto. Portanto,
não puseram a Jesus, por causa da preparação dos judeus ', para o
túmulo estava perto. João 19:41-42
A preparação aqui mencionado foi o sexto dia da semana, porque o dia
seguinte era uma alta sábado. Mas a preparação urgente aqui é duplo.
Aqui, novamente, a palavra "dia" foi acrescentada pelos tradutores. O
primeiro é a preparação para a Festa da Páscoa, que começou ao pôr
do sol do dia 14. A Festa da Páscoa dura do pôr ao nascer do sol na
manhã seguinte do dia 15. O sábado do sétimo dia não era para
começar até o amanhecer do dia seguinte, altura em que a Festa dos
Pães Ázimos começou bem. Alta sábado era o termo usado quando
uma festa sábado anual nomeado pousou em um sábado do sétimo dia
regular. Isso ocorre apenas duas vezes por ano, e neste caso, foi o
primeiro dia da Festa dos Pães Ázimos.
Além disso, observe a última frase. Ele abençoou o dia de sábado eo
separou como sagrado e santo. Páscoa é o único lugar na Bíblia onde o
escuro da noite é designado como A luz era para simbolizar a verdade
ea justiça. Não há inconsistências em relação a este simbolismo
ordenado. O sábado era para ser apenas as horas do dia. Era para ser
de crepúsculo até o crepúsculo, que é o verdadeiro significado de "a
partir mesmo até mesmo". Levítico 23:05, 32.
Abaixo está um relógio de sol grego antigo, que, por sua concepção está
em harmonia com as Escrituras (João 11:9). Embora este relógio de sol
está faltando seu "gnomon" (ponteiro) observar os números são da
esquerda para a direita, começando no número 1 e terminando no
número 12. Isso demonstra que a primeira hora após o nascer do sol é
a primeira hora do dia. Ele também demonstra que a 6 ª hora do dia é
meio-dia, e 12 ª hora termina precisamente pôr do sol. Se é verão ou
inverno sempre há 12 horas em um dia. Relógios de sol só medir raios
diretos do sol, e não medem a noite, ou qualquer parte do anoitecer ou
de madrugada. É o projeto original do relógio que determina os
parâmetros mensuráveis de um dia, separando o dia da noite. Em
contraste marcante, os relógios de sol gregorianos romanas seguir um
sistema numérico que exibe a 12 ª hora, como o meio-dia,
correlacionando-se com um dia de 24 horas que começa à meia-noite.
De acordo com a Escritura, do meio-dia era a sexta hora do dia.
Alfabeto grego e do sistema numérico:
Pergunta 40: Durante a vida do Messias Ele guardou o sábado para o
comprimento correto de tempo. Se Israel foi mantê-lo errado,
certamente ele teria corrigido eles. Quanto tempo durou o Messias
declarar um "dia" de ser, o que também se aplica ao dia do sábado do
sétimo dia?
Jesus respondeu: "Não são doze as horas do dia? Se alguém andar de
dia, não tropeça, porque vê a luz deste mundo." João 11:09 NVI
Resposta: Enquanto Ele afirma que o dia seja 12 horas de duração,
observe Ele esclarece nenhuma diferença entre o comprimento de um
dia da semana e um dia de sábado. Desta forma, Ele os define como o
mesmo. Os judeus tradicionais não são capazes de utilizar esse detalhe,
porque a não reconhecer ou o Messias ou o Novo Testamento.ia
quarenta noites e quarenta dias.
Pergunta 41: Além disso, durante a vida do Messias que era seu
costume ir à sinagoga. Esteve presente durante a parte clara ou escura
de uma data do calendário lunar?
E, como seu costume, ele entrou na sinagoga no dia de sábado, e
levantou-se para ler. Lucas 4:16 NTLH
Resposta: De acordo com a Escritura que era seu costume de entrar na
sinagoga durante a luz do dia. No onde está registrado que ele foi à
sinagoga para o culto durante a escuridão da noite. Os judeus
tradicionais não são capazes de utilizar esse detalhe.
Pergunta 42: O Messias, Yahushua era o Cordeiro sacrificial que deu a
sua vida como oferta pelo pecado, em nome da humanidade. Depois de
um julgamento de agonia e espancamento, nosso Salvador foi colocado
na cruz na sexta hora. Isso levanta a questão, foi esta sexta hora do dia,
ou a sexta hora da noite?
Agora era o dia da preparação da Páscoa, e cerca da hora sexta. João
19:14 NVI
Resposta: Quando este afirma que era a "hora sexta", então você deve
perceber que este tempo contínuo é contado a partir do nascer do sol,
fazendo nascer no início de cada novo dia. Como resultado, o Salvador
teria sido colocado na cruz, às 12h00. No entanto, se você insistir o dia
começa na "noite", porque você acredita que o "dia" começou a
contar a partir do sol, então você também deve acreditar que o
Salvador foi crucificado em cerca de 12:00 à meia-noite e morreu por
volta de 03h00 (Gregoriano tempo), durante a escuridão da noite. Você
não pode ter as duas coisas.
Como um prego num lugar firme, um eclipse solar ocorreu no dia da
crucificação, do sexto ao nono hora. Os eclipses solares ocorrem
apenas durante o dia, quando o sol é totalmente visível.
Veja gráfico:
Pergunta 43: Páscoa, Abib 14, AD 31, foi a data do calendário lunar da
crucificação. Este dia de dias, também conhecido como o dia da
preparação, continha apenas 12 horas de sol a sol. Quantas horas do
dia seguinte contém?
Agora era o dia da preparação da Páscoa, e cerca da hora sexta. João
19:14 NVI
Agora, a partir da sexta hora até a hora nona, houve trevas sobre toda
a terra. Mateus 27:45-46
Agora, quando a sexta hora tinha chegado, houve trevas sobre toda a
terra até a hora nona. Mark 15:33-34
O Salvador foi colocado na cruz na sexta hora do dia. No momento da
sexta hora (meio-dia) havia trevas sobre toda a terra até a hora nona
(3:00 PM gregoriano Time). Lucas 23:45 esclarece, ainda, que isso foi
causado pelo sol escurecido.
Um sol escurecerá é sinônimo de um eclipse solar. Para 4000 anos
todos os eclipses solares ocorreu no dia 14 ou 15 do mês lunar, e este
mês não foi excepção ao sistema de tempo do Criador. Pelo contrário,
era o cumprimento de todas as maneiras, e com destaque para um
comprimento sobrenatural das trevas, como a atenção de todo o céu
estava voltada para a Terra e do Filho morrendo de Yahuah. Consulte
o estudo da palavra, "O sol escureceu da sexta para a Nona Hora".
Resposta: O dia seguinte continha exatamente a mesma quantidade de
horas, o que foi um total de doze. O dia seguinte à crucificação, não era
apenas um sábado do sétimo dia, mas também foi o primeiro dia da
Festa dos Pães Ázimos, tornando-se uma alta sábado. O número de
horas sagradas para este dia era exatamente o mesmo que as horas
sagradas do sétimo dia da criação. A parte sagrada do sábado do
sétimo dia tem sido sempre apenas as horas do dia, e nunca foi para
incluir a escuridão da noite. Os judeus tradicionais não são capazes de
utilizar esse detalhe.
Pergunta 44: "nascer do sol, até o seu ocaso" Se a celebração do nosso
dia de sábado começa quando o sol está se pondo, por que o nosso
estado Criador que todas as nações louvarão o Seu nome e adorá-lo a
partir do
Desde o nascer do sol até o seu ocaso nome de Yahuah é para ser
elogiado. Salmos 113:3 NVI
O Poderoso, [Elohiym] [Yahuah], falou e chamou a terra desde o
nascer do sol até o seu ocaso. Desde Sião, a perfeição da beleza,
[Elohiym] brilhará. Salmos 50:1-2 NVI
Mas desde o nascente do sol, até o seu ocaso, meu nome é grande entre
os gentios, em todo lugar incenso deve ser oferecido ao meu nome, e
uma oblação pura; porque o meu nome é grande entre as nações ", diz
[Yahuah] dos Exércitos. Malaquias 1:11 NTLH
Resposta: Se o dia de sábado começa ao pôr do sol ou na primeira luz
da aurora, como muitas pessoas acreditam, então estes versos não faria
sentido algum. A Escritura define claramente que o louvor do Todo-
Poderoso [Elohiym] do Céu é feito de sol a sol.
Recap:
A primeira coisa que o Criador chamou à existência era "luz". Gênesis
1:3.A escuridão não foi criado pelo Criador Yahuah, mas sim que é a
ausência de luz e Seu poder criativo. Gênesis 1:2-4.
Um dia é a parte "luz do sol" de uma data do calendário lunar total,
ocupando o período de sol a sol e é definido pelo número de horas em
um relógio de sol. Gênesis 1:4, 5.
A noite ocupa a porção do pôr ao nascer do sol, de uma data do
calendário lunar completo, e é definida por quatro relógios. Gênesis
1:4-5.
Dia e noite são períodos opostos do tempo que nunca se sobrepõem.
Gênesis 1:4, 5.
Uma distinção que separa estava sempre lançar quando o Criador
chamado a luz do dia ea escuridão da noite. Gênesis 1:4-5.
Dia e noite são as duas metades de uma data do calendário lunar
completo. Gênesis 1:4, 5.
Noite é equiparada com a mistura de luz conhecido como "Twilight"
ou "troca de luz", que ocorre pouco antes do nascer do sol e logo após
o pôr do sol. Gênesis 1:05.
"O dia começa ao nascer do sol e termina ao pôr do sol e totaliza um
período de doze horas. João 11:09.
Toda a criação foi realizada durante as horas de luz do dia. exceto
quando a luz foi chamado diante das trevas no primeiro dia. Gênesis 1:
todos.
As quatro vigílias da noite ocupam o período entre o pôr eo nascer do
sol, como a duração da noite é definida pelas estrelas visíveis. Jeremias
31:35-36
As regras visíveis sol e as medidas a dia, enquanto as estrelas visíveis
governar e medir a noite. O dia ea noite nunca pode sobrepor-se, mais
do que o brilho do sol e as estrelas podem se sobrepor ou ser visíveis ao
mesmo tempo. Jeremias 31:35-36
O Criador declarou seis vezes em seis dias que Suas obras de criação
foram realizadas durante o dia, que foram compostas da noite seguinte
pela manhã. Gênesis 1:1-31.
O sol foi nomeado para governar o dia. Gênesis 1:16.
A lua e as estrelas foram nomeados para governar a noite. Gênesis
1:16.
Yahuah sempre usou os princípios contrastantes de luz e escuridão
para ensinar suas verdades espirituais. Provérbios 4:18-19, João 8:12,
Mateus 5:14, 15; João 1:4, 7; João 3:20, 1 Pedro 2:09, Efésios 5:13.
Metaforicamente, "escuridão" foi para representar a infidelidade, a
cegueira espiritual e do mal. Provérbios 4:18-19, João 1:04, João 3:20;
1 Pedro 2:9.
Metaforicamente, "luz" foi para representar verdade, compreensão
espiritual e justiça. Provérbios 4:18-19, João 1:04, João 3:20; 1 Pedro
2:9.
Os termos de dia ou amanhã seguinte, ambos referem-se à luz do dia,
que começa ao nascer do sol e termina ao pôr do sol. Êxodo 18:13.
Hipócritas chamar o dia da noite. Isso é demonstrado por todos
aqueles que acreditam que o santo sábado do sétimo dia começa ao pôr
do sol. Job 17:8-14.
Se o decreto do dia e portaria da noite são removidos, então Israel
devia cessar como uma nação. Job 17:8-14.
"Os termos" mesmo até mesmo "se referem ao nascer ao pôr como"
mesmo "representa a" troca de luz ", e foi criada como suportes de
livros metafóricos para um" dia ". Levítico 23:05, 32.
O Quarto Mandamento suporta o que foi descoberto em Gênesis 1:1-5,
que o sábado é a porção à luz do dia na data do calendário lunar
completo. Não há absolutamente nenhuma menção à escuridão da
noite no quarto mandamento. Consulte o estudo da palavra, Êxodo
20:7-11 " A Lua Nova Beacon Encontrado no Terceiro e Quarto
Mandamento ".
O Criador abençoou o dia de sábado eo separou como sagrado e santo.
Em nenhum lugar na Bíblia é a escuridão da noite já designado como
sagrado e santo do que outro, possivelmente, a Festa da Páscoa, pois só
naquela noite foi divinamente projetado para simbolizar a cegueira
espiritual e do mal. No caso da Páscoa, era o sangue do Cordeiro de
[Elohiym] que era resgatar a humanidade pecadora e as trevas do
mundo, caiu em pecado. Provérbios 4:18-19, João 1:04, João 3:20; 1
Pedro 2:9.
O dia de sábado é o mesmo período de tempo, como todos os outros
dias, incorporando um período de 12 horas de luz do dia. Gênesis 1:1-
5; Gênesis 2:1-3, João 11:09.
Durante a vida do Messias que era seu costume de entrar na sinagoga,
no "dia" de sábado e nunca a "noite". Lucas 4:16.
Durante a vida do Messias Ele declarou que há apenas "12 horas" em
um dia. Afinal de contas, Ele também foi o criador, que chamou o
"dia" de luz e às trevas chamou "noite". Ele não muda. João 11:09;
Eclesiastes 3:14, Isaías 40:8.
Em Sua morte Ele confirmou que o dia está apenas doze horas de
duração. Ele foi crucificado na terceira hora do "dia". O céu ficou
escuro desde a hora sexta (meio-dia) até a hora nona, Ele declarou,
está consumado e morreu. Lembre-se da noite não é dividido por
horas, portanto isso está se referindo especificamente para as horas do
dia.
Yahuah declarou que as nações dos gentios da terra iria adorá-lo desde
o nascer do sol para ele vai para baixo. Salmos 113:3, Salmos 50:1-2;
Malaquias 1:11.
Que você seja abundantemente abençoado em tudo que você faz, como
você continuar olhando para cima e buscando o Pai das luzes (Tiago
1:17). -
Fonte: Kerry Frances
http://www.thecreatorscalendar.com/Articles/When_does_a_day_begi
n/07_When_does_a_day_begin.html

Mais conteúdo relacionado

Mais procurados

Sábado Semanal ou Sábado Lunar?
Sábado Semanal ou Sábado Lunar?Sábado Semanal ou Sábado Lunar?
Sábado Semanal ou Sábado Lunar?ASD Remanescentes
 
O verdadeiro sabado é o sabado semanal e não o sabado lunar
O verdadeiro sabado é o sabado semanal e não o sabado lunarO verdadeiro sabado é o sabado semanal e não o sabado lunar
O verdadeiro sabado é o sabado semanal e não o sabado lunarASD Remanescentes
 
Luna Nueva
Luna NuevaLuna Nueva
Luna Nuevaantso
 
A teologia de satanás nos últimos dias
A teologia de satanás nos últimos diasA teologia de satanás nos últimos dias
A teologia de satanás nos últimos diasEduardo Sousa Gomes
 
Estrutura do santuário
Estrutura do santuárioEstrutura do santuário
Estrutura do santuárioJosé Santos
 
Teologia do antigo testamento a esperança messianica
Teologia do antigo testamento a esperança messianicaTeologia do antigo testamento a esperança messianica
Teologia do antigo testamento a esperança messianicaJose Ventura
 
O Shabbat Lunar em Êxodo 12, será? Refutado Serie 03
O Shabbat Lunar em Êxodo 12, será? Refutado Serie 03O Shabbat Lunar em Êxodo 12, será? Refutado Serie 03
O Shabbat Lunar em Êxodo 12, será? Refutado Serie 03ASD Remanescentes
 
TEXTOS ADULTERADOS DA BIBLIA
TEXTOS ADULTERADOS DA BIBLIATEXTOS ADULTERADOS DA BIBLIA
TEXTOS ADULTERADOS DA BIBLIAASD Remanescentes
 
Lição 3 - A Natureza dos Demônios – Agentes da Maldade no Mundo Espiritual
Lição 3 - A Natureza dos Demônios – Agentes da Maldade no Mundo EspiritualLição 3 - A Natureza dos Demônios – Agentes da Maldade no Mundo Espiritual
Lição 3 - A Natureza dos Demônios – Agentes da Maldade no Mundo EspiritualHamilton Souza
 
Mateus 28:19 em Hebraico e não em Grego - Estudo Completo
Mateus  28:19 em Hebraico e não em Grego  - Estudo CompletoMateus  28:19 em Hebraico e não em Grego  - Estudo Completo
Mateus 28:19 em Hebraico e não em Grego - Estudo CompletoASD Remanescentes
 
A Divisão das horas do dia nos tempos Bíblicos
A Divisão das horas do dia nos tempos BíblicosA Divisão das horas do dia nos tempos Bíblicos
A Divisão das horas do dia nos tempos BíblicosASD Remanescentes
 
1 el ruaj ha kodesh 1ra. parte
1 el ruaj ha kodesh 1ra. parte1 el ruaj ha kodesh 1ra. parte
1 el ruaj ha kodesh 1ra. parteBeit meshobeb TX
 
El nacimiento de yeshúa y sukot
El nacimiento de yeshúa y sukotEl nacimiento de yeshúa y sukot
El nacimiento de yeshúa y sukotpublicador
 
La Muerte Y Resurreccion De Yahshua El Mesias 1224987081662800 9
La Muerte Y Resurreccion De Yahshua El Mesias 1224987081662800 9La Muerte Y Resurreccion De Yahshua El Mesias 1224987081662800 9
La Muerte Y Resurreccion De Yahshua El Mesias 1224987081662800 9DANIEL RIVERO
 
Takanot - portugues - final (1).pdf
Takanot - portugues - final (1).pdfTakanot - portugues - final (1).pdf
Takanot - portugues - final (1).pdfPatriciaMorgado10
 
MILÊNIO, JUÍZO FINAL E ETERNIDADE
MILÊNIO, JUÍZO FINAL E ETERNIDADEMILÊNIO, JUÍZO FINAL E ETERNIDADE
MILÊNIO, JUÍZO FINAL E ETERNIDADELeonam dos Santos
 

Mais procurados (20)

Sábado Semanal ou Sábado Lunar?
Sábado Semanal ou Sábado Lunar?Sábado Semanal ou Sábado Lunar?
Sábado Semanal ou Sábado Lunar?
 
O verdadeiro sabado é o sabado semanal e não o sabado lunar
O verdadeiro sabado é o sabado semanal e não o sabado lunarO verdadeiro sabado é o sabado semanal e não o sabado lunar
O verdadeiro sabado é o sabado semanal e não o sabado lunar
 
Luna Nueva
Luna NuevaLuna Nueva
Luna Nueva
 
70 semanas-de-daniel
70 semanas-de-daniel70 semanas-de-daniel
70 semanas-de-daniel
 
O ano de jubileu
O ano de jubileuO ano de jubileu
O ano de jubileu
 
Yahushua
YahushuaYahushua
Yahushua
 
A teologia de satanás nos últimos dias
A teologia de satanás nos últimos diasA teologia de satanás nos últimos dias
A teologia de satanás nos últimos dias
 
Estrutura do santuário
Estrutura do santuárioEstrutura do santuário
Estrutura do santuário
 
Teologia do antigo testamento a esperança messianica
Teologia do antigo testamento a esperança messianicaTeologia do antigo testamento a esperança messianica
Teologia do antigo testamento a esperança messianica
 
O Shabbat Lunar em Êxodo 12, será? Refutado Serie 03
O Shabbat Lunar em Êxodo 12, será? Refutado Serie 03O Shabbat Lunar em Êxodo 12, será? Refutado Serie 03
O Shabbat Lunar em Êxodo 12, será? Refutado Serie 03
 
TEXTOS ADULTERADOS DA BIBLIA
TEXTOS ADULTERADOS DA BIBLIATEXTOS ADULTERADOS DA BIBLIA
TEXTOS ADULTERADOS DA BIBLIA
 
Lição 3 - A Natureza dos Demônios – Agentes da Maldade no Mundo Espiritual
Lição 3 - A Natureza dos Demônios – Agentes da Maldade no Mundo EspiritualLição 3 - A Natureza dos Demônios – Agentes da Maldade no Mundo Espiritual
Lição 3 - A Natureza dos Demônios – Agentes da Maldade no Mundo Espiritual
 
Mateus 28:19 em Hebraico e não em Grego - Estudo Completo
Mateus  28:19 em Hebraico e não em Grego  - Estudo CompletoMateus  28:19 em Hebraico e não em Grego  - Estudo Completo
Mateus 28:19 em Hebraico e não em Grego - Estudo Completo
 
Deuteronomio 3
Deuteronomio 3Deuteronomio 3
Deuteronomio 3
 
A Divisão das horas do dia nos tempos Bíblicos
A Divisão das horas do dia nos tempos BíblicosA Divisão das horas do dia nos tempos Bíblicos
A Divisão das horas do dia nos tempos Bíblicos
 
1 el ruaj ha kodesh 1ra. parte
1 el ruaj ha kodesh 1ra. parte1 el ruaj ha kodesh 1ra. parte
1 el ruaj ha kodesh 1ra. parte
 
El nacimiento de yeshúa y sukot
El nacimiento de yeshúa y sukotEl nacimiento de yeshúa y sukot
El nacimiento de yeshúa y sukot
 
La Muerte Y Resurreccion De Yahshua El Mesias 1224987081662800 9
La Muerte Y Resurreccion De Yahshua El Mesias 1224987081662800 9La Muerte Y Resurreccion De Yahshua El Mesias 1224987081662800 9
La Muerte Y Resurreccion De Yahshua El Mesias 1224987081662800 9
 
Takanot - portugues - final (1).pdf
Takanot - portugues - final (1).pdfTakanot - portugues - final (1).pdf
Takanot - portugues - final (1).pdf
 
MILÊNIO, JUÍZO FINAL E ETERNIDADE
MILÊNIO, JUÍZO FINAL E ETERNIDADEMILÊNIO, JUÍZO FINAL E ETERNIDADE
MILÊNIO, JUÍZO FINAL E ETERNIDADE
 

Destaque

As Horas do Dia e o inicio do Sábado.
As Horas do Dia e o inicio do Sábado.As Horas do Dia e o inicio do Sábado.
As Horas do Dia e o inicio do Sábado.ASD Remanescentes
 
Iasd rejeitará o sábado (3)
Iasd rejeitará o sábado (3)Iasd rejeitará o sábado (3)
Iasd rejeitará o sábado (3)Jose Moraes
 
1948 - Será uma coincidência ou Obra de Deus?
1948 - Será uma coincidência ou Obra de Deus?1948 - Será uma coincidência ou Obra de Deus?
1948 - Será uma coincidência ou Obra de Deus?Helio Colombe
 
16 compreendendo as trombetas. apoc. 8, 9
16   compreendendo as trombetas. apoc. 8, 916   compreendendo as trombetas. apoc. 8, 9
16 compreendendo as trombetas. apoc. 8, 9Diego Fortunatto
 
Esse poderia ser o dia que o apóstolo viu no Apocalipse!!! 2017
Esse poderia ser o dia que o apóstolo viu no Apocalipse!!! 2017Esse poderia ser o dia que o apóstolo viu no Apocalipse!!! 2017
Esse poderia ser o dia que o apóstolo viu no Apocalipse!!! 2017Helio Colombe
 
Agentes Infiltrados na IASD
Agentes Infiltrados na IASDAgentes Infiltrados na IASD
Agentes Infiltrados na IASDJosé Silva
 
A origem oculta dos símolos na igreja adventista
A origem oculta dos símolos na igreja adventistaA origem oculta dos símolos na igreja adventista
A origem oculta dos símolos na igreja adventistaEduardo Sousa Gomes
 
What Makes Great Infographics
What Makes Great InfographicsWhat Makes Great Infographics
What Makes Great InfographicsSlideShare
 
Masters of SlideShare
Masters of SlideShareMasters of SlideShare
Masters of SlideShareKapost
 
STOP! VIEW THIS! 10-Step Checklist When Uploading to Slideshare
STOP! VIEW THIS! 10-Step Checklist When Uploading to SlideshareSTOP! VIEW THIS! 10-Step Checklist When Uploading to Slideshare
STOP! VIEW THIS! 10-Step Checklist When Uploading to SlideshareEmpowered Presentations
 
10 Ways to Win at SlideShare SEO & Presentation Optimization
10 Ways to Win at SlideShare SEO & Presentation Optimization10 Ways to Win at SlideShare SEO & Presentation Optimization
10 Ways to Win at SlideShare SEO & Presentation OptimizationOneupweb
 
How To Get More From SlideShare - Super-Simple Tips For Content Marketing
How To Get More From SlideShare - Super-Simple Tips For Content MarketingHow To Get More From SlideShare - Super-Simple Tips For Content Marketing
How To Get More From SlideShare - Super-Simple Tips For Content MarketingContent Marketing Institute
 
How to Make Awesome SlideShares: Tips & Tricks
How to Make Awesome SlideShares: Tips & TricksHow to Make Awesome SlideShares: Tips & Tricks
How to Make Awesome SlideShares: Tips & TricksSlideShare
 

Destaque (17)

As Horas do Dia e o inicio do Sábado.
As Horas do Dia e o inicio do Sábado.As Horas do Dia e o inicio do Sábado.
As Horas do Dia e o inicio do Sábado.
 
Nibiru
NibiruNibiru
Nibiru
 
O shabat biblico
O shabat biblicoO shabat biblico
O shabat biblico
 
Iasd rejeitará o sábado (3)
Iasd rejeitará o sábado (3)Iasd rejeitará o sábado (3)
Iasd rejeitará o sábado (3)
 
1948 - Será uma coincidência ou Obra de Deus?
1948 - Será uma coincidência ou Obra de Deus?1948 - Será uma coincidência ou Obra de Deus?
1948 - Será uma coincidência ou Obra de Deus?
 
16 compreendendo as trombetas. apoc. 8, 9
16   compreendendo as trombetas. apoc. 8, 916   compreendendo as trombetas. apoc. 8, 9
16 compreendendo as trombetas. apoc. 8, 9
 
Esse poderia ser o dia que o apóstolo viu no Apocalipse!!! 2017
Esse poderia ser o dia que o apóstolo viu no Apocalipse!!! 2017Esse poderia ser o dia que o apóstolo viu no Apocalipse!!! 2017
Esse poderia ser o dia que o apóstolo viu no Apocalipse!!! 2017
 
A BíBlia E As Estrelas
A BíBlia E As EstrelasA BíBlia E As Estrelas
A BíBlia E As Estrelas
 
Agentes Infiltrados na IASD
Agentes Infiltrados na IASDAgentes Infiltrados na IASD
Agentes Infiltrados na IASD
 
A origem oculta dos símolos na igreja adventista
A origem oculta dos símolos na igreja adventistaA origem oculta dos símolos na igreja adventista
A origem oculta dos símolos na igreja adventista
 
What Makes Great Infographics
What Makes Great InfographicsWhat Makes Great Infographics
What Makes Great Infographics
 
Masters of SlideShare
Masters of SlideShareMasters of SlideShare
Masters of SlideShare
 
STOP! VIEW THIS! 10-Step Checklist When Uploading to Slideshare
STOP! VIEW THIS! 10-Step Checklist When Uploading to SlideshareSTOP! VIEW THIS! 10-Step Checklist When Uploading to Slideshare
STOP! VIEW THIS! 10-Step Checklist When Uploading to Slideshare
 
You Suck At PowerPoint!
You Suck At PowerPoint!You Suck At PowerPoint!
You Suck At PowerPoint!
 
10 Ways to Win at SlideShare SEO & Presentation Optimization
10 Ways to Win at SlideShare SEO & Presentation Optimization10 Ways to Win at SlideShare SEO & Presentation Optimization
10 Ways to Win at SlideShare SEO & Presentation Optimization
 
How To Get More From SlideShare - Super-Simple Tips For Content Marketing
How To Get More From SlideShare - Super-Simple Tips For Content MarketingHow To Get More From SlideShare - Super-Simple Tips For Content Marketing
How To Get More From SlideShare - Super-Simple Tips For Content Marketing
 
How to Make Awesome SlideShares: Tips & Tricks
How to Make Awesome SlideShares: Tips & TricksHow to Make Awesome SlideShares: Tips & Tricks
How to Make Awesome SlideShares: Tips & Tricks
 

Semelhante a Por que o sábado não começa ao pôr do sol

Horário para o início do sábado j. n. andrews
Horário para o início do sábado   j. n. andrewsHorário para o início do sábado   j. n. andrews
Horário para o início do sábado j. n. andrewsASD Remanescentes
 
A formação do mundo_Lição da Escola Sabatina_original_com_textos
A formação do mundo_Lição da Escola Sabatina_original_com_textos A formação do mundo_Lição da Escola Sabatina_original_com_textos
A formação do mundo_Lição da Escola Sabatina_original_com_textos Gerson G. Ramos
 
009-Jornal Cesesul 25-05-14
009-Jornal Cesesul 25-05-14009-Jornal Cesesul 25-05-14
009-Jornal Cesesul 25-05-14CESESUL
 
Gêneses 1:14 e o Shabbat Lunar - Refutado Serie 02
Gêneses 1:14 e o Shabbat Lunar - Refutado Serie 02 Gêneses 1:14 e o Shabbat Lunar - Refutado Serie 02
Gêneses 1:14 e o Shabbat Lunar - Refutado Serie 02 ASD Remanescentes
 
Expressão lição-09 2016 2ºtrimestre
Expressão   lição-09 2016 2ºtrimestreExpressão   lição-09 2016 2ºtrimestre
Expressão lição-09 2016 2ºtrimestreJoel Silva
 
O verdadeiro 7º dia (parte 3) quando começa o dia
O verdadeiro 7º dia (parte 3)   quando começa o diaO verdadeiro 7º dia (parte 3)   quando começa o dia
O verdadeiro 7º dia (parte 3) quando começa o diaEfraim Ben Tzion
 
O JULGAMENTO FINAL: Qual será a sua sorte: gozo eterno ou desprezo eterno?
O JULGAMENTO FINAL: Qual será a sua sorte: gozo eterno ou desprezo eterno?O JULGAMENTO FINAL: Qual será a sua sorte: gozo eterno ou desprezo eterno?
O JULGAMENTO FINAL: Qual será a sua sorte: gozo eterno ou desprezo eterno?Carlos Oliveira
 
(08) apresentacão aula 8 a luz do primeiro dia
(08) apresentacão aula 8 a luz do primeiro dia(08) apresentacão aula 8 a luz do primeiro dia
(08) apresentacão aula 8 a luz do primeiro diaGlauciaSlides
 
Lição 2 a criação dos céus e da terra
Lição 2   a criação dos céus e da terraLição 2   a criação dos céus e da terra
Lição 2 a criação dos céus e da terraAndrew Guimarães
 
Jesus, Criador do céu e da Terra_Lição da Escola Sabatina_original_com_textos
 Jesus, Criador do céu e da Terra_Lição da Escola Sabatina_original_com_textos  Jesus, Criador do céu e da Terra_Lição da Escola Sabatina_original_com_textos
Jesus, Criador do céu e da Terra_Lição da Escola Sabatina_original_com_textos Gerson G. Ramos
 
Deus é Luz
Deus é LuzDeus é Luz
Deus é Luzlucena
 
Estudos Bíblicos - Gênesis em perguntas e respostas
Estudos Bíblicos - Gênesis em perguntas e respostasEstudos Bíblicos - Gênesis em perguntas e respostas
Estudos Bíblicos - Gênesis em perguntas e respostasContos e Estudos Bíblicos
 
A formação do mundo_Resumo_Liç_212013_Esc_Sab.
A formação do mundo_Resumo_Liç_212013_Esc_Sab.A formação do mundo_Resumo_Liç_212013_Esc_Sab.
A formação do mundo_Resumo_Liç_212013_Esc_Sab.Gerson G. Ramos
 
HISTÓRIAS BÍBLICAS PARA CRIANÇAS - O LIVRO DE GÊNESISDisponível em: www.porta...
HISTÓRIAS BÍBLICAS PARA CRIANÇAS - O LIVRO DE GÊNESISDisponível em: www.porta...HISTÓRIAS BÍBLICAS PARA CRIANÇAS - O LIVRO DE GÊNESISDisponível em: www.porta...
HISTÓRIAS BÍBLICAS PARA CRIANÇAS - O LIVRO DE GÊNESISDisponível em: www.porta...leitemel
 
PÔR DO SOL - MARAVILHA DE DEUS
PÔR DO SOL - MARAVILHA DE DEUSPÔR DO SOL - MARAVILHA DE DEUS
PÔR DO SOL - MARAVILHA DE DEUSESCRIBAVALDEMIR
 
Ebd adbvv-genesis-licao 2
Ebd adbvv-genesis-licao 2Ebd adbvv-genesis-licao 2
Ebd adbvv-genesis-licao 2Figueiredo Marc
 
Universo Criado - versao curta 20min
Universo Criado - versao curta 20minUniverso Criado - versao curta 20min
Universo Criado - versao curta 20minAndré Luiz Marques
 

Semelhante a Por que o sábado não começa ao pôr do sol (20)

Horário para o início do sábado j. n. andrews
Horário para o início do sábado   j. n. andrewsHorário para o início do sábado   j. n. andrews
Horário para o início do sábado j. n. andrews
 
A formação do mundo_Lição da Escola Sabatina_original_com_textos
A formação do mundo_Lição da Escola Sabatina_original_com_textos A formação do mundo_Lição da Escola Sabatina_original_com_textos
A formação do mundo_Lição da Escola Sabatina_original_com_textos
 
009-Jornal Cesesul 25-05-14
009-Jornal Cesesul 25-05-14009-Jornal Cesesul 25-05-14
009-Jornal Cesesul 25-05-14
 
Gêneses 1:14 e o Shabbat Lunar - Refutado Serie 02
Gêneses 1:14 e o Shabbat Lunar - Refutado Serie 02 Gêneses 1:14 e o Shabbat Lunar - Refutado Serie 02
Gêneses 1:14 e o Shabbat Lunar - Refutado Serie 02
 
Expressão lição-09 2016 2ºtrimestre
Expressão   lição-09 2016 2ºtrimestreExpressão   lição-09 2016 2ºtrimestre
Expressão lição-09 2016 2ºtrimestre
 
Doutrina da Criação
Doutrina da CriaçãoDoutrina da Criação
Doutrina da Criação
 
O verdadeiro 7º dia (parte 3) quando começa o dia
O verdadeiro 7º dia (parte 3)   quando começa o diaO verdadeiro 7º dia (parte 3)   quando começa o dia
O verdadeiro 7º dia (parte 3) quando começa o dia
 
O JULGAMENTO FINAL: Qual será a sua sorte: gozo eterno ou desprezo eterno?
O JULGAMENTO FINAL: Qual será a sua sorte: gozo eterno ou desprezo eterno?O JULGAMENTO FINAL: Qual será a sua sorte: gozo eterno ou desprezo eterno?
O JULGAMENTO FINAL: Qual será a sua sorte: gozo eterno ou desprezo eterno?
 
Deus é luz
Deus é luzDeus é luz
Deus é luz
 
(08) apresentacão aula 8 a luz do primeiro dia
(08) apresentacão aula 8 a luz do primeiro dia(08) apresentacão aula 8 a luz do primeiro dia
(08) apresentacão aula 8 a luz do primeiro dia
 
Lição 2 a criação dos céus e da terra
Lição 2   a criação dos céus e da terraLição 2   a criação dos céus e da terra
Lição 2 a criação dos céus e da terra
 
Jesus, Criador do céu e da Terra_Lição da Escola Sabatina_original_com_textos
 Jesus, Criador do céu e da Terra_Lição da Escola Sabatina_original_com_textos  Jesus, Criador do céu e da Terra_Lição da Escola Sabatina_original_com_textos
Jesus, Criador do céu e da Terra_Lição da Escola Sabatina_original_com_textos
 
Deus é Luz
Deus é LuzDeus é Luz
Deus é Luz
 
A criação dos céus, e da terr
A criação dos céus, e da terrA criação dos céus, e da terr
A criação dos céus, e da terr
 
Estudos Bíblicos - Gênesis em perguntas e respostas
Estudos Bíblicos - Gênesis em perguntas e respostasEstudos Bíblicos - Gênesis em perguntas e respostas
Estudos Bíblicos - Gênesis em perguntas e respostas
 
A formação do mundo_Resumo_Liç_212013_Esc_Sab.
A formação do mundo_Resumo_Liç_212013_Esc_Sab.A formação do mundo_Resumo_Liç_212013_Esc_Sab.
A formação do mundo_Resumo_Liç_212013_Esc_Sab.
 
HISTÓRIAS BÍBLICAS PARA CRIANÇAS - O LIVRO DE GÊNESISDisponível em: www.porta...
HISTÓRIAS BÍBLICAS PARA CRIANÇAS - O LIVRO DE GÊNESISDisponível em: www.porta...HISTÓRIAS BÍBLICAS PARA CRIANÇAS - O LIVRO DE GÊNESISDisponível em: www.porta...
HISTÓRIAS BÍBLICAS PARA CRIANÇAS - O LIVRO DE GÊNESISDisponível em: www.porta...
 
PÔR DO SOL - MARAVILHA DE DEUS
PÔR DO SOL - MARAVILHA DE DEUSPÔR DO SOL - MARAVILHA DE DEUS
PÔR DO SOL - MARAVILHA DE DEUS
 
Ebd adbvv-genesis-licao 2
Ebd adbvv-genesis-licao 2Ebd adbvv-genesis-licao 2
Ebd adbvv-genesis-licao 2
 
Universo Criado - versao curta 20min
Universo Criado - versao curta 20minUniverso Criado - versao curta 20min
Universo Criado - versao curta 20min
 

Mais de Efraim Ben Tzion

Mais de Efraim Ben Tzion (20)

Adela
AdelaAdela
Adela
 
As 2 faces de yeshua
As 2 faces de yeshuaAs 2 faces de yeshua
As 2 faces de yeshua
 
Quem éo principe da paz
Quem éo principe da pazQuem éo principe da paz
Quem éo principe da paz
 
A biblia de yeshua
A biblia de yeshuaA biblia de yeshua
A biblia de yeshua
 
Quem é satan
Quem é satanQuem é satan
Quem é satan
 
Elohim plural ou singular
Elohim plural ou singularElohim plural ou singular
Elohim plural ou singular
 
Satanas expulsa satanas
Satanas expulsa satanasSatanas expulsa satanas
Satanas expulsa satanas
 
O que há de errado com a idolatria
O que há de errado com a idolatriaO que há de errado com a idolatria
O que há de errado com a idolatria
 
NAO é Necessario um sacrifico com sangue
NAO é Necessario um sacrifico com sangueNAO é Necessario um sacrifico com sangue
NAO é Necessario um sacrifico com sangue
 
Jesus ou barrabas a farsa
Jesus ou barrabas   a farsaJesus ou barrabas   a farsa
Jesus ou barrabas a farsa
 
Diabo no imaginário cristão -ebook
Diabo no imaginário cristão -ebookDiabo no imaginário cristão -ebook
Diabo no imaginário cristão -ebook
 
Isaias 7
Isaias 7Isaias 7
Isaias 7
 
Salmo 2 vs 7
Salmo 2 vs 7Salmo 2 vs 7
Salmo 2 vs 7
 
Rei ezequias ou o falso messias
Rei ezequias ou o falso messiasRei ezequias ou o falso messias
Rei ezequias ou o falso messias
 
O sacrificio e o perdão
O sacrificio e o perdãoO sacrificio e o perdão
O sacrificio e o perdão
 
Profecia de miqueias
Profecia de miqueiasProfecia de miqueias
Profecia de miqueias
 
A fé de abrãao contra paulo
A fé de abrãao contra pauloA fé de abrãao contra paulo
A fé de abrãao contra paulo
 
Quem alterou o salmo 110
Quem alterou o salmo 110Quem alterou o salmo 110
Quem alterou o salmo 110
 
O que jeremias disse sobre nova aliança
O que jeremias disse sobre nova aliançaO que jeremias disse sobre nova aliança
O que jeremias disse sobre nova aliança
 
A arvore sem logica de jc
A arvore sem logica de jcA arvore sem logica de jc
A arvore sem logica de jc
 

Último

As festas esquecidas.pdf................
As festas esquecidas.pdf................As festas esquecidas.pdf................
As festas esquecidas.pdf................natzarimdonorte
 
Tabela bíblica Periódica - Livros da Bíblia.pdf
Tabela bíblica Periódica - Livros da Bíblia.pdfTabela bíblica Periódica - Livros da Bíblia.pdf
Tabela bíblica Periódica - Livros da Bíblia.pdfAgnaldo Fernandes
 
Vivendo a vontade de Deus para adolescentes - Cleide Silva
Vivendo a vontade de Deus para adolescentes - Cleide SilvaVivendo a vontade de Deus para adolescentes - Cleide Silva
Vivendo a vontade de Deus para adolescentes - Cleide SilvaSammis Reachers
 
toaz.info-livro-a-trilha-menos-percorrida-de-m-scott-peck-compressed-pr_8b78f...
toaz.info-livro-a-trilha-menos-percorrida-de-m-scott-peck-compressed-pr_8b78f...toaz.info-livro-a-trilha-menos-percorrida-de-m-scott-peck-compressed-pr_8b78f...
toaz.info-livro-a-trilha-menos-percorrida-de-m-scott-peck-compressed-pr_8b78f...VANESSACABRALDASILVA
 
Oração A Bem-Aventurada Irmã Dulce Dos Pobres
Oração A Bem-Aventurada Irmã Dulce Dos PobresOração A Bem-Aventurada Irmã Dulce Dos Pobres
Oração A Bem-Aventurada Irmã Dulce Dos PobresNilson Almeida
 
As violações das leis do Criador (material em pdf)
As violações das leis do Criador (material em pdf)As violações das leis do Criador (material em pdf)
As violações das leis do Criador (material em pdf)natzarimdonorte
 
EUBIOSOFIA - MEMÓRIAS DA SOCIEDADE TEOSÓFICA BRASILEIRA
EUBIOSOFIA - MEMÓRIAS DA SOCIEDADE TEOSÓFICA BRASILEIRAEUBIOSOFIA - MEMÓRIAS DA SOCIEDADE TEOSÓFICA BRASILEIRA
EUBIOSOFIA - MEMÓRIAS DA SOCIEDADE TEOSÓFICA BRASILEIRAMarco Aurélio Rodrigues Dias
 
METODOLOGIA ELANA* – ENSINO LEVA AUTONOMIA NO APRENDIZADO. UMA PROPOSTA COMP...
METODOLOGIA ELANA* – ENSINO LEVA  AUTONOMIA NO APRENDIZADO. UMA PROPOSTA COMP...METODOLOGIA ELANA* – ENSINO LEVA  AUTONOMIA NO APRENDIZADO. UMA PROPOSTA COMP...
METODOLOGIA ELANA* – ENSINO LEVA AUTONOMIA NO APRENDIZADO. UMA PROPOSTA COMP...PIB Penha
 
Dar valor ao Nada! No Caminho da Autorrealização
Dar valor ao Nada! No Caminho da AutorrealizaçãoDar valor ao Nada! No Caminho da Autorrealização
Dar valor ao Nada! No Caminho da Autorrealizaçãocorpusclinic
 
O Livro dos Mortos do Antigo Egito_240402_210013.pdf
O Livro dos Mortos do Antigo Egito_240402_210013.pdfO Livro dos Mortos do Antigo Egito_240402_210013.pdf
O Livro dos Mortos do Antigo Egito_240402_210013.pdfmhribas
 
Série Evangelho no Lar - Pão Nosso - Cap. 132 - Em tudo
Série Evangelho no Lar - Pão Nosso - Cap. 132 - Em tudoSérie Evangelho no Lar - Pão Nosso - Cap. 132 - Em tudo
Série Evangelho no Lar - Pão Nosso - Cap. 132 - Em tudoRicardo Azevedo
 
Lição 4 - Como se Conduzir na Caminhada.pptx
Lição 4 - Como se Conduzir na Caminhada.pptxLição 4 - Como se Conduzir na Caminhada.pptx
Lição 4 - Como se Conduzir na Caminhada.pptxCelso Napoleon
 
Paulo é vítima de fake news e o primeiro culto num domingo
Paulo é vítima de fake news e o primeiro culto num domingoPaulo é vítima de fake news e o primeiro culto num domingo
Paulo é vítima de fake news e o primeiro culto num domingoPIB Penha
 
Lição 5 - Os Inimigos do Cristão - EBD.pptx
Lição 5 - Os Inimigos do Cristão - EBD.pptxLição 5 - Os Inimigos do Cristão - EBD.pptx
Lição 5 - Os Inimigos do Cristão - EBD.pptxCelso Napoleon
 

Último (16)

VICIOS MORAIS E COMPORTAMENTAIS NA VISÃO ESPÍRITA
VICIOS MORAIS E COMPORTAMENTAIS  NA VISÃO ESPÍRITAVICIOS MORAIS E COMPORTAMENTAIS  NA VISÃO ESPÍRITA
VICIOS MORAIS E COMPORTAMENTAIS NA VISÃO ESPÍRITA
 
As festas esquecidas.pdf................
As festas esquecidas.pdf................As festas esquecidas.pdf................
As festas esquecidas.pdf................
 
Tabela bíblica Periódica - Livros da Bíblia.pdf
Tabela bíblica Periódica - Livros da Bíblia.pdfTabela bíblica Periódica - Livros da Bíblia.pdf
Tabela bíblica Periódica - Livros da Bíblia.pdf
 
Vivendo a vontade de Deus para adolescentes - Cleide Silva
Vivendo a vontade de Deus para adolescentes - Cleide SilvaVivendo a vontade de Deus para adolescentes - Cleide Silva
Vivendo a vontade de Deus para adolescentes - Cleide Silva
 
toaz.info-livro-a-trilha-menos-percorrida-de-m-scott-peck-compressed-pr_8b78f...
toaz.info-livro-a-trilha-menos-percorrida-de-m-scott-peck-compressed-pr_8b78f...toaz.info-livro-a-trilha-menos-percorrida-de-m-scott-peck-compressed-pr_8b78f...
toaz.info-livro-a-trilha-menos-percorrida-de-m-scott-peck-compressed-pr_8b78f...
 
Centros de Força do Perispírito (plexos, chacras)
Centros de Força do Perispírito (plexos, chacras)Centros de Força do Perispírito (plexos, chacras)
Centros de Força do Perispírito (plexos, chacras)
 
Oração A Bem-Aventurada Irmã Dulce Dos Pobres
Oração A Bem-Aventurada Irmã Dulce Dos PobresOração A Bem-Aventurada Irmã Dulce Dos Pobres
Oração A Bem-Aventurada Irmã Dulce Dos Pobres
 
As violações das leis do Criador (material em pdf)
As violações das leis do Criador (material em pdf)As violações das leis do Criador (material em pdf)
As violações das leis do Criador (material em pdf)
 
EUBIOSOFIA - MEMÓRIAS DA SOCIEDADE TEOSÓFICA BRASILEIRA
EUBIOSOFIA - MEMÓRIAS DA SOCIEDADE TEOSÓFICA BRASILEIRAEUBIOSOFIA - MEMÓRIAS DA SOCIEDADE TEOSÓFICA BRASILEIRA
EUBIOSOFIA - MEMÓRIAS DA SOCIEDADE TEOSÓFICA BRASILEIRA
 
METODOLOGIA ELANA* – ENSINO LEVA AUTONOMIA NO APRENDIZADO. UMA PROPOSTA COMP...
METODOLOGIA ELANA* – ENSINO LEVA  AUTONOMIA NO APRENDIZADO. UMA PROPOSTA COMP...METODOLOGIA ELANA* – ENSINO LEVA  AUTONOMIA NO APRENDIZADO. UMA PROPOSTA COMP...
METODOLOGIA ELANA* – ENSINO LEVA AUTONOMIA NO APRENDIZADO. UMA PROPOSTA COMP...
 
Dar valor ao Nada! No Caminho da Autorrealização
Dar valor ao Nada! No Caminho da AutorrealizaçãoDar valor ao Nada! No Caminho da Autorrealização
Dar valor ao Nada! No Caminho da Autorrealização
 
O Livro dos Mortos do Antigo Egito_240402_210013.pdf
O Livro dos Mortos do Antigo Egito_240402_210013.pdfO Livro dos Mortos do Antigo Egito_240402_210013.pdf
O Livro dos Mortos do Antigo Egito_240402_210013.pdf
 
Série Evangelho no Lar - Pão Nosso - Cap. 132 - Em tudo
Série Evangelho no Lar - Pão Nosso - Cap. 132 - Em tudoSérie Evangelho no Lar - Pão Nosso - Cap. 132 - Em tudo
Série Evangelho no Lar - Pão Nosso - Cap. 132 - Em tudo
 
Lição 4 - Como se Conduzir na Caminhada.pptx
Lição 4 - Como se Conduzir na Caminhada.pptxLição 4 - Como se Conduzir na Caminhada.pptx
Lição 4 - Como se Conduzir na Caminhada.pptx
 
Paulo é vítima de fake news e o primeiro culto num domingo
Paulo é vítima de fake news e o primeiro culto num domingoPaulo é vítima de fake news e o primeiro culto num domingo
Paulo é vítima de fake news e o primeiro culto num domingo
 
Lição 5 - Os Inimigos do Cristão - EBD.pptx
Lição 5 - Os Inimigos do Cristão - EBD.pptxLição 5 - Os Inimigos do Cristão - EBD.pptx
Lição 5 - Os Inimigos do Cristão - EBD.pptx
 

Por que o sábado não começa ao pôr do sol

  • 1. 44 RAZÕES PORQUE O SABADO NÃO COMEÇA AO POR-DO-SOL Segundo o peso da evidência nas Escrituras, um dia é de apenas 12 horas de duração e começa ao nascer do sol e termina ao pôr do sol. A noite é dividida por quatro relógios com início ao pôr do sol e terminando ao amanhecer. A data do calendário completo é 24 horas de comprimento e não é sinônimo de um dia. Em contraste uma data do calendário civil plena inclui todos os componentes de ambos o dia ea noite, e começa ao amanhecer e não à meia-noite ou pôr do sol. Um dia é apenas a primeira metade de uma data do calendário lunar completo, enquanto a noite é a segunda metade. O dia ea noite ocupar porções opostas de tempo em uma data do calendário lunar completo. Dia e noite NUNCA sobrepõem ou ocupar o mesmo espaço de tempo. Para as pessoas muito tempo, boas e honestas têm se contentado em seguir fábulas judaicas e romanas (Tito 1:14) para quando o dia começa que não são suportados pelo peso da evidência nas Escrituras. No entanto, ao longo de toda a verdade foi lá para a tomada, com um pouco de esforço para reunir todas as provas do original hebraico ou palavras gregas. O simples facto de a esta hora tardia esta unidade de tempo está sendo revelado, deve dizer-nos o nosso Messias está voltando em breve para
  • 2. seus fiéis, que será totalmente restaurada para a cronometragem de Seu reino. A seguir, a verdade presente é projetado para equipar fiel de Yahuah para as batalhas mais verdade que temos pela frente. Pergunta 1: De acordo com Gênesis 1:1-5, qual foi a primeira coisa que o Criador convocado à existência? No começo [Elohiym] criou os céus ea terra. A terra era sem forma e vazia, e havia trevas sobre a face do abismo. E o Espírito de [Elohiym] pairava sobre a face das águas. Então [Elohiym] disse: "Haja luz", e houve luz. E [Elohiym] viu a luz que era bom, e [Elohiym] separação entre a luz e as trevas. [Elohiym] chamou à luz Dia e às trevas chamou Noite. Gênesis 1:1-5 NVI Resposta: A primeira coisa que Ele criou era "luz".
  • 3. Pergunta 2: Qual foi a primeira coisa que o Criador chamado? No começo [Elohiym] criou os céus ea terra. A terra era sem forma e vazia, e havia trevas sobre a face do abismo. E o Espírito de [Elohiym] pairava sobre a face das águas. Então [Elohiym] disse: "Haja luz", e houve luz. E [Elohiym] viu a luz, que era bom, e [Elohiym] separação entre a luz e as trevas. [Elohiym] chamou à luz dia, e às trevas chamou Noite. Gênesis 1:1-5 NVI Resposta: Luz foi a primeira coisa que o Criador nomeado. Ele chamou o "dia". Luz Questão 3: Qual foi a segunda coisa que o Criador chamado? No começo [Elohiym] criou os céus ea terra. A terra era sem forma e vazia, e havia trevas sobre a face do abismo. E o Espírito de [Elohiym] pairava sobre a face das águas. Então [Elohiym] disse: "Haja luz", e houve luz. E [Elohiym] viu a luz, que era bom, e [Elohiym] separação entre a luz e as trevas. [Elohiym] chamou à luz dia, e às trevas chamou Noite. Gênesis 1:1-5 NVI Resposta: A escuridão era a segunda coisa que o Criador nomeado. Ele chamou das trevas "noite". Pergunta 4: De acordo com Gênesis 1:1-5, quando foi que o Criador fazer a escuridão? No começo [Elohiym] criou os céus ea terra a terra era sem forma e vazia;. E havia trevas sobre a face do abismo. E o Espírito de [Elohiym] pairava sobre a face das águas. Então [Elohiym] disse: "Haja luz", e houve luz. E [Elohiym] viu a luz, que era bom, e [Elohiym] separação entre a luz e as trevas. [Elohiym] chamou à luz dia, e às trevas chamou Noite. Gênesis 1:1-5 NVI Resposta: Por mais surpreendente que isso possa ser para alguns, o Criador não fazer ou criar a escuridão. Como definido acima, a escuridão é a ausência de luz. A escuridão é a ausência da força criadora. Para fazer uma distinção entre a escuridão ea luz do criador separados deles, o que significa que eles vão sempre atuar em diferentes períodos de tempo e nunca se sobrepõem. Ele, então, selou sua declaração de que Ele os separou quando Ele deu-lhes os seus
  • 4. nomes. Dia e noite, não são termos sinônimos, mas sim que eles são opostos. Pergunta 5: Uma vez que a escuridão é a ausência de luz, então o que é a noite? Resposta: a noite é a ausência do dia, tal como definido por Genesis 1:1-5. Questão 6: De acordo com o Novo Testamento inteiro, a luz do dia é dividido em 12 unidades básicas de tempo. O que são chamadas essas unidades? E quando veio aqueles que foram contratados cerca da hora undécima, receberam um denário cada um. Mateus 20:9-10 NVI Jesus respondeu: "Não são doze as horas do dia? Se alguém andar de dia, não tropeça, porque vê a luz deste mundo. 10 Mas, se alguém andar de noite, tropeça, porque a luz é não nele ". João 11:9-11 NVI Resposta: Essas unidades são chamadas de "horas". A luz do dia, de sol a sol, é composta por doze horas. Horas foram contados pelas sombras em um relógio de sol. Mesmo que os dias de verão são mais longos do que os dias de inverno, ele sempre aparece em um relógio de sol como 12 horas. Pergunta 7: Um relógio funciona somente durante as horas de sol a sol, então o que foi usado para contar as unidades de tempo durante a escuridão da noite? Resposta: É os celestes estrelas / planetas nos céus acima que determinar o comprimento das quatro vigílias da noite (Marcos 13:35). Noite Watch - Esta primeira vigília da noite começa ao pôr do sol e ocupou o período conhecido como entardecer ou crepúsculo, a mistura de luz e trevas, e termina na escuridão completa. Mateus 13:35
  • 5. Meia-noite Watch - Esta segunda vigília da noite começa às trevas completo e termina precisamente à meia-noite. Era a posição da lua e as estrelas que declararam verdadeiro meia-noite. Lucas 0:38 Galo cantando Watch - Esta terceira vigília da noite começa à meia- noite e termina na primeira luz da aurora ou crepúsculo da manhã. Além disso, este foi o período em que era característico de quando o galo começou a cantar. Lucas 0:38 Manhã Watch - Esta quarta vigília da noite, começa na primeira luz visível no céu do leste, o tempo que o galo começou a cantar, o que também é conhecido como aurora ou crepúsculo da manhã, a mistura de luz e escuridão. A manhã termina Assista ao nascer do sol. Mateus 14:25 Pergunta 8: Existe uma aplicação secundária, o que tornaria a divisão do "dia ea" noite ", uma lei espiritual do Criador. Resposta: De acordo com a Escritura, a noite foi dividida em quatro relógios. A lua, juntamente com a posição das estrelas, foi o guia para conhecer cada quarta parte da noite. Observe como esses parâmetros da noite também definem os limites do "dia". Não pode haver sobreposição. A noite termina como o novo dia começa ao amanhecer. O termo "turno da noite" nasceu da idéia de ver os primeiros raios de um novo dia ao amanhecer. O foco estava sempre na luz, procurando a luz, sempre buscando a luz. Desta forma, uma ilustração viva foi criado que expressa os lugares valor Yahuah sobre a luz do dia, em contraste com a escuridão da noite.Porque todo aquele que faz o mal odeia a luz e não vem para a luz, para que as suas obras não sejam reprovadas. Mas quem pratica a verdade vem para a luz que as suas obras sejam manifestas, porque são feitas em [Elohiym]. João 3:20-21 Mas, se os teus olhos forem maus, todo o teu corpo será tenebroso. Portanto, caso a luz que em ti há são trevas, quão grandes serão tais trevas! Ninguém pode servir a dois senhores, porque ou há de odiar a um e amar o outro, ou se dedicará a um e desprezará o outro. Não podeis servir a [Elohiym] ea Mamom. Mateus 6:23-24
  • 6. Porque todos vós sois filhos da luz e filhos do dia: nós não somos da noite nem das trevas. Portanto, não durmamos, como os demais, mas vigiemos e sejamos sóbrios. 1 Tessalonicenses 5:5-6 Mais uma vez, um novo mandamento que vos escrevo, o qual é verdadeiro nele e em vós, porque as trevas se vão dissipando, ea verdadeira luz já brilha. Aquele que diz que está na luz, e odeia a seu irmão, está nas trevas até agora. Aquele que ama a seu irmão permanece na luz, e não há nenhuma causa para tropeço nele. Mas aquele que odeia a seu irmão está nas trevas e anda nas trevas, e não sabe para onde vai, porque as trevas lhe cegaram os olhos. 1 João 2:8- 11 NVI Pergunta 9: Se Yahuah dá o sol para luz do dia, e as estrelas para luz da noite, então como pode o dia começa ao pôr do sol ou anoitecer, à meia-noite, de manhã crepúsculo antes do nascer do sol, ou ao meio- dia? Há pessoas ao redor do mundo que reconheceram esses períodos como o início de seu "dia" e, especificamente, em nome de seu santo dia. Resposta: De acordo com Jeremias 31:35, quando a primeira estrela é testemunhado no céu, podemos saber que a "noite" já começou. Enquanto o próprio sol é visível, o que significa nascer ao pôr do sol, é "dia em que o sol está no poder." Assim diz Yahuah, que dá o sol para luz do dia, as ordenanças da lua e das estrelas para luz da noite. "Se estas ordenanças de diante de mim, diz Yahuah, então a semente de Israel deixam também de ser uma nação diante de mim para sempre." Jeremias 31:35-36 NTLH A partir deste versículo podemos adquirir os seguintes fatos: Quando o versículo acima fala das estrelas que está falando de estrelas visíveis. Quando o versículo acima fala do sol o mesmo princípio aplica-se, por isso, fala de um sol visível. Quando as estrelas estão presentes e visíveis, é "noite". Quando o sol está presente e visível, é "dia".
  • 7. O sol e as estrelas nunca estão visíveis ao mesmo tempo. A lua visível também é falado, embora a segunda metade do mês a lua nem sempre é visível à noite, durante as fases lunares diminuindo. "Esta é a instrução direta pelo Criador, Yahuah, para a divisão do dia e da noite. Dia e noite estão aqui diferenciados especificamente pelo "sol" e as "estrelas". Eles simplesmente não são visíveis ao mesmo tempo. Quando o sol deixa de brilhar as estrelas começam o seu resplendor. Por favor, note que não é apenas a luz que é dado para a parte que compõe o "dia", mas afirma o "sol" é para luz do dia. Metade do mês a lua é visível à noite e metade do mês, é visível a dia. Mas as estrelas nunca variam sobre o seu resplendor. A palavra "decreto", aqui, significa o " tempo determinado circuito do sol, a lua e as estrelas. " É a lua e as estrelas que, especificamente, dar luz à noite. Ao pôr do sol de um ou dois planetas / estrelas começam a ser visíveis, e uma ou duas estrelas continuam a ser visível até o amanhecer. Se o dia começou ao meio-dia, sol, ou meia-noite, ele faria com que o sol, a lua e as estrelas para se sobrepor. Gênesis 1:14-18 define o parâmetro de dia e noite com a simples declaração de que o sol estava para governar o dia, ea lua e as estrelas foram para governar a noite. Quanto mais claro o Pai Celestial poderia ser? Se o "dia" ocupou o período desde o amanhecer até o final do entardecer, o "dia" também estaria sobrepondo o brilho das estrelas ea lua para 1 ½ a 2 horas ao anoitecer e, em seguida, novamente, pouco antes do nascer do sol. Nós podemos saber com certeza que o período imediatamente antes do nascer do sol ainda é considerado "noite" pelo Criador, bem como logo após pôr do sol, porque há pelo menos um planeta / estrela visível pouco antes do nascer do sol, e pelo menos um planeta / estrela visível apenas após o ocaso. Foi o Criador, mesmo quem esclareceu que a divisão entre "dia" e "noite" se manifesta pelo sol visível, a lua e as estrelas. Quando o "dia" é dividido corretamente, não haverá sobreposição de suas luzes celestiais. Portanto, a única divisão possível do dia e da noite é de sol a sol, já que não há sobreposição de luz do sol com a lua e as estrelas. No momento em que os raios do sol radiantes desaparecer ao pôr do sol as
  • 8. primeiras estrelas começam a aparecer em perfeita harmonia com a divisão do dia e da noite, no Gênesis 1 e Êxodo conta 16. Consulte o artigo, " estrelas visíveis definir os parâmetros de dia e noite . " Pergunta 10: De acordo com Gênesis 1:5, quando é que uma noite começa e termina? [Elohiym] chamou à luz dia, e às trevas chamou Noite. Gênesis 1:1-5 NVI Resposta: A noite começa ao pôr do sol como este é quando a escuridão se mistura com a luz. A noite termina ao nascer do sol do amanhecer seguinte mistura de luz e escuridão. Então noite ocupa a segunda parcela de uma data do calendário lunar logo após pôr do sol, quando as estrelas começam um a um, para tornar-se visivelmente presente. Pergunta 11: Assim, se uma noite começa ao pôr do sol, ocupando todo o período negro, quando é que um dia começa? [Elohiym] chamou à luz dia, e às trevas chamou Noite. Gênesis 1:1-5 NVI Resposta: O dia foi contada apenas por as horas do dia, e começou ao amanhecer e terminou ao pôr do sol. Quando as pessoas declaram que o "dia" começa ao pôr do sol, eles estão realmente dizendo que "a luz é igual a escuridão." Pergunta 12: De acordo com Gênesis 1:5, que é a definição dada para a palavra "noite?" [Elohiym] chamou à luz dia, e às trevas chamou Noite. Assim foi a tarde ea manhã, o dia primeiro. Gênesis 01:05 NVI Resposta: O Criador ordenou que a luz é igual a dia e que a escuridão é igual a noite, mas agora somos confrontados com o termo "noite", que não é esclarecido em Gênesis 1, como o que ela é. . Utilize dois ou mais léxicos hebraicos como foi feito abaixo para saber os significados da palavra hebraica ('Ereb # 6153). Definição palavra "noite" (Ereb # 6153) # 1689 Theological Wordbook do Antigo Testamento (TWOT) corresponde a # 6153 na Concordância de Strong.
  • 9. 'Ereb: Noite, noite. Provavelmente desenvolvido a partir da expressão ", o pôr do sol, pôr do sol." Inclui: para entrar, para ir para baixo do sol. . . 'Ereb é encontrado 131 vezes no AT. A frase "não havia uma noite e havia uma manhã" ocorre seis vezes na narrativa da criação (Gênesis 1:5-31), delimitando os seis dias de atividade criativa divina. Esta frase parece indicar que no antigo Israel um dia começou com nascer do sol. Alguns sentiram esta em desacordo com a prática judaica de considerar pôr do sol como o início do dia seguinte. Cassuto, depois de lidar com os dados bíblicos eo costume judaico, conclui que não era "apenas um sistema de tempo de computação: o dia é considerado para começar no dia ... Este julgamento parece justificado no emprego de" Ereb na legislação levítico respeitando a imundícia. Uma era considerada impura por causa de certos atos "até a noite" (Levítico 11:24, mais de trinta vezes). Isto é, um era imundo para a duração do dia. Teológico Wordbook do Antigo Testamento (TWOT) # 1689, p. 694. Significa: anoitecer, noite, noite, pôr do sol. No estudo palavra abaixo, mostra-se que o termo "noite" é igual ao momento do pôr do sol, o início da escuridão da noite, também conhecida como a "troca de crepúsculo." As "trocas de crepúsculo" ocorrem durante a noite, tanto o amanhecer eo entardecer, marcando o início eo fim do do Criador "noite". Stro ng # King James Versio n da Bíblia Hebra ico Strong Definições Brown-driver- Briggs Hebrew Lexicon Novo Significado Comprehensive Erros evidentes na tradução e adicionado # 9999 palavras são destacadas neste verde Versículo 5 7121 E qara ' para chamar;endereço pelonome; convidar; pregar, proclamar; pronunciar; publicar; falar; chamar, a chamar, a recitar, para ler, para gritar, para proclamar, proclamar; aonom e, para dar nome a, para Dirigindo-lo pelo nome, chamado
  • 10. ser nomeado, a ser escolhido; 430 [Elohiy m] 'Elohiy m nome plural de [Elohiym], referindo-se especificamente ao supremo [Elohiym];'Elohiym; 'Elohiym (plural) governantes, juízes; os divinos; anjos; [Elohiym] s; (significado intensivo singular plural) [Elohiym]; o único e verdadeiro [Elohiym]; Elohiym, 216 a luz 'Owr iluminação; brilhante; luz; a luz do dia, a luz de luminares celestes (a Lua, o Sol, as estrelas); luz dodia; madrugada, a luz da manhã. a luz 3117 dia yowm de uma raiz não utilizada significando ser quente; um dia (como as horasquentes), sol a sol, pôr do sol ao pôr do sol; (Nota:. Pôr do sol a sol não se pode aplicar aqui, porque o processo de criação foi especificamente concebido para separar a luz das trevas, o dia da noite, e não incluir a noite do dia) dia; tempo; ano;por dia, em oposição anoite, um dia de trabalho, uma divisão de tempo; "Dia" em oposição à noite (as horas quentes do nascer ao pôr do sol), 2822 ea escurid ão choshe k escuro; trevas,trevas; noite; trevas, trevas; ea escuridão 7121 chama do qara para chamar;endereço pelo nome; convidar; pregar, proclamar; pronunciar; publicar; falar; chamar, a chamar, a recitar, para ler, para gritar, para proclamar, proclamar; ao nome, para dar nome a, para ser nomeado, a ser Chamou
  • 11. escolhido; 3915 noite. layil noite, aescuridão,estação noite; um toque longe da luz; temporada noite. Nota: Noite não podem ser incluídos no termo "dia", porque o Criador está aqui fazendo uma distinção entre "dia e noite", separando-os, Ele não é colocá-los juntos. noite (em oposição ao dia) Nota: Este Léxico hebraico define noite como distintamente separada do "dia". Um princípio fundamental é definido aqui em pedra pelo Criador, a Si mesmo que o dia ea noite devem ser para sempre e distintamente separada. "Noite", em oposição ao dia(o período escuro do pôr ao nascer do sol). 1961 e Hayah de existir, ou tornar- se, vir apassar, farol, completamente; seguir, aconteceu; ser, para se tornar, a acontecer, paraexi stir, a acontecer, para cair; Depois veio 6153 a noite `Ereb `Ereb # 6154 - Palavr a - Root # 6148 `árabe anoitecer; maré da noite, noite,dia mistura trançar, intermix; se misturam; dar penhores; envolver, dar para ser de segurança (como uma espécie detroca); pôr do sol,noite, noite ; # 6154 `Ereb - que é a mesma palavra hebraica: trama, como misto;entrel açados; material de malha; mistura; pessoas mistos; sociedade de economia mista. a troca, para envolver, dar em penhor, para assumir um compromisso, para garantir; o anoitecer(Crepú sculo)
  • 12. Palavr a Raiz - # 6150 `árabe através da idéia de cobrir com textura; sol; serescurecido; para troca de promessas; ter comunhão com; 1961 e hayah de existir, ou tornar-se, vir a passar, farol, completamente;seguir,a conteceu; ser, para se tornar, a acontecer, paraexi stir, a acontecer, para cair; em seguida, veio 1242 Pela manhã Boqer madrugada,manhã, rom per do dia; manhã, final da noite, vinda daluz do dia;vinda donascer do sol;início do dia;alegria figurativamente brilhante depois de uma noite de angústia, o dia de amanhã, no dia seguinte, na manhã seguinte; o amanhecer, (Crepúsculo) 3117 dia. yowm de uma raiz não utilizada significando ser quente; um dia (como as horasquentes), sol a sol, pôr do sol ao pôr do sol; (Nota:. Pôr do sol a sol não se pode aplicar aqui, porque o processo de criação foi especificamente concebido para separar a luz das trevas, o dia da noite, e não incluir a noite do dia) dia; tempo; ano;por dia, em oposição anoite, um dia de trabalho, uma divisão de tempo; dia (em oposição a noite, as horas quentes do nascer ao pôr do sol) 259 o primeir 'Echad um numeral que significa unificar; um, um (número), cada, cada, um um.
  • 13. o como umprimeiro ordinal; certo, apenas, uma vez que, uma vez por todas, em primeiro lugar; Dirigindo-lo pelo nome, Elohiym chamou à luz dia, ao contrário de noite, e às trevas chamou noite, ao contrário do dia. Depois veio o anoitecer (crepúsculo), depois veio o amanhecer (crepúsculo), um dia. Êxodo 01:05 Pergunta 13: De acordo com a tabela de Gênesis 1:05 palavra estudo acima, se o Criador dizer que a noite faz parte do dia ou da parte da noite? Resposta: De acordo com o estudo da palavra hebraica "noite" (Ereb # 6153) significa pôr do sol, noite, e ao anoitecer. Cada um destes são equiparados a noite, desde a noite começa ao pôr do sol. Pergunta 14: As modernas versões da Bíblia afirmar que "tarde ea manhã" foram o primeiro dia, segundo dia, etc, isso não significa que "a noite vem antes do amanhecer?" Resposta: Na superfície este parece ser o caso, no entanto, uma rápida olhada em qualquer concordância ou léxico proporcionará ampla evidência de que duas palavras foram mal traduzidas. A palavra hebraica em questão aqui é (hayah # 1961), o que significa: existir, tornar-se, acontecer, vir a passar, siga, (ver tabela abaixo). Mas em vez de qualquer uma dessas opções ou um sinônimo estes, as palavras "so" e "e" foram aplicadas em seu lugar, fazendo parecer que a noite anterior à luz do dia. Strong # King James Version da Bíblia Hebraico Strong Definições Brown-driver- Briggs Hebrew Lexicon Novo Significado Comprehensive
  • 14. 1961 e hayah de existir, ou tornar-se, vir a passar, farol, completamente;seguir,aconteceu; ser, para se tornar, aacontecer, para existir, a acontecer, para cair; em seguida, veio ou seguido Aqui está como o texto deve ser: Dirigindo-lo pelo nome, Elohiym chamou à luz dia, ao contrário de noite, e às trevas chamou noite, ao contrário do dia. Depois veio o anoitecer (crepúsculo), depois veio o amanhecer (crepúsculo), um dia. Êxodo 01:05
  • 15. Pergunta 15: Agora, com uma compreensão mais completa das diferenças do dia e da noite, como declarado pelo Criador, durante o período de tempo que Ele fez todas as Suas obras da criação? E a terra produziu erva, a erva que dá semente segundo a sua espécie, ea árvore que dá fruto, cuja semente está nela conforme a sua espécie. E [Elohiym] viu que era bom. Depois veio o anoitecer (crepúsculo), depois veio o amanhecer (crepúsculo), dia três. Gênesis 1:12-13 Resposta: O foco da criação foi a beleza do que é visível na luz, então parece que todas as obras criativas foram realizadas durante o tempo da luz maravilhosa de as horas de luz do nascer ao pôr do sol. Absolutamente nada foi criado durante a escuridão da noite, após o Criador provocou a "luz". Aqui abaixo é um gráfico simples delinear círculo diário da criação do tempo, como é descrito na Bíblia Criação do Diário Círculo do Tempo DIA ONE 1. Sol a sol, em que foram feitas as obras criativas - Yahuah convocado luz ea divisão do dia e da noite foram estabelecidas. 2. Depois veio o anoitecer (crepúsculo), 3. Depois veio o amanhecer (crepúsculo), um dia. SEGUNDO DIA 1. Sol a sol, em que foram feitas as obras criativas - Yahuah dividiu as águas acima e abaixo do firmamento. 2. Depois veio o anoitecer (crepúsculo), 3. Depois veio o amanhecer (crepúsculo), dia dois.
  • 16. TERCEIRO DIA 1. Sol a sol, em que foram feitas as obras criativas - Yahuah trouxe terra seca, grama, ervas e sementes de rolamento. 2. Depois veio o anoitecer (crepúsculo), 3. Depois veio o amanhecer (crepúsculo), dia três, etc, etc Pergunta 16: Em Gênesis 1, Moisés registra o Criador declarando seis vezes acima do círculo de tempo, um dia, dois, três, etc Então, quando o sétimo dia é mencionado no Capítulo 2, o período de tempo que o Criador designar como Santo? Era um círculo completo de tempo, a luz do dia, seguido pela noite, ou foi apenas para ser a luz do dia? E no sétimo dia [Elohiym] terminou a obra que tinha feito, descansou no sétimo dia de toda a Sua obra, que tinha feito. 3 Então [Elohiym] abençoou o sétimo dia eo santificou, porque nele descansou de toda a obra que [Elohiym] criara e fizera. Gênesis 2:2-3 NVI Resposta: As horas sagradas do sábado são apenas a parte à luz do dia o círculo completo de 24 horas do tempo, e são contados por horas. Em nenhum lugar nas Escrituras há uma pitada de vigílias da noite que ocorrem no sábado. Se o dia estava para começar ao pôr do sol, este versículo da Bíblia seria o lugar onde seria abertamente declarado pelo Criador, afirmando que ele descansou no sétimo noite, três vezes, em vez do sétimo dia. Veja João 11:09. A verdade fundamental estabelecido em Gênesis 01:05 foi a divisão permanente da luz do dia da noite. Toda a criação foi feito durante a luz do dia, por isso é lógico que, quando o Criador cessado ou descansou no sétimo dia, foi durante o mesmo período de tempo que Ele anteriormente fez seu trabalho de criar.
  • 17. Pergunta 17: Que exemplos das Escrituras mostram que o Pai Celestial projetou as realidades físicas da luz e das trevas com uma aplicação simbólica espiritual? Resposta: Em toda a Escritura Yahuah usou esses princípios contrastantes de luz e escuridão para ensinar suas verdades espirituais Trevas era representar metaforicamente infidelidade, cegueira espiritual e do mal, enquanto a luz era representar a verdade, a compreensão espiritual e justiça.. Yahushua usa esses princípios mais e mais ao longo de seu ministério junto com Paul.Mas a vereda dos justos é como a luz da aurora, que vai brilhando mais e mais até ser dia perfeito. O caminho dos ímpios é como a escuridão: não sabem eles em que tropeçam. Provérbios 4:18-19 Yahushua, o Messias disse: "Eu sou a luz do mundo: quem me segue não andará em trevas, mas terá a luz da vida." João 8:12 "Vós sois a luz do mundo". Mateus 5:14 "Deixe sua luz brilhe diante dos homens" Mateus 5:16 Nele estava a vida, ea vida era a luz dos homens. E a luz resplandece nas trevas e as trevas não a compreenderam. João 1:4 Este veio como testemunha, para dar testemunho da luz que todos os homens cressem por meio dele. João 1:7 "Porque todo aquele que faz o mal odeia a luz, e não vem para a luz, para que as suas obras não sejam reprovadas. Mas aquele que faz, vem para a luz, para que as suas obras sejam manifestas, porque são feitas em [Elohiym] . " João 3:20 Mas vós sois a geração eleita, o sacerdócio real, a nação santa, o povo adquirido, para que vos anunciar as virtudes daquele que tinha chamou das trevas para a sua maravilhosa luz. 1Pedro 2:09 Mas todas estas coisas, sendo condenadas, se manifestam pela luz, pois tudo o que se manifesta é luz. Efésios 5:13 Pergunta 18: O que Yahuah, nosso Criador nos dá para determinar os nossos dias e os nossos anos, luzes ou escuridão?
  • 18. Então [Elohiym] disse: "Haja luzeiros no firmamento dos céus para separar o dia da noite, e sejam eles para sinais e para estações, e para dias e anos Gênesis 1:14 NTLH. Toda boa dádiva e todo dom perfeito são lá do alto, descendo do Pai das luzes, em quem não pode existir variação ou sombra de mudança. Tiago 1:17 NTLH Resposta: Luzes! Observe que não diz noites e as trevas, mas as luzes foram dadas para separar o dia da noite. O "dia" é regido pelos raios diretos do sol, ocupando o tempo do nascer ao pôr do sol. As luzes da noite ligar quando o pôr do sol, como uma a uma as estrelas começam a brilhar. Além disso, o nosso Criador é referido como o "Pai das Luzes", em que não há sombra de uma órbita de viragem. O sol e as estrelas nunca tem sombras, então esta referência de "sem sombra" é só para a lua. Quando a lua representa o "Pai das Luzes" é somente quando não há sombra de sua órbita de viragem. Há apenas uma fase lunar que se encaixa nessa descrição, e que é a "cheia de" Lua Nova. Louve Yahuah! Pergunta 19: Se um "dia" começa ao pôr do sol, quando se torna escuro, então quando é que uma "noite" começa? Quando os gazeu foi dito: "Sansão chegou aqui!" eles cercaram o local e ficaram à espera dele a noite na porta da cidade. Eles ficaram quietos a noite toda, dizendo: "Na parte da manhã, quando é a luz do dia, vamos matá-lo." Juízes 16:02 NVI Fazes as trevas, e vem a noite, na qual todos os animais da floresta sobre fluência. Salmos 104:20 NKJV Enquanto a terra durar, sementeira e colheita, frio e calor, verão e inverno, e de dia e de noite
  • 19. não cessarão. Gênesis 8:22 KJV Resposta: Dia e noite não tanto iniciar simultaneamente no mesmo momento. Se assim for, que seria a mesma coisa. Um dia começa quando ela se torna a luz do dia (raios diretos do sol) de acordo com Juízes 16:02. Noite começa quando se torna escuro, o tempo em que os animais começam a rastejar sobre o acordo com o Salmo 104:20. O mais impressionante dos exemplos acima é a terceira em Gênesis 8:22, que ilustra esse ponto com quatro comparações únicas. Cada uma destas comparações são opostos totais encontrados na natureza, ilustrando o dia ea noite como o epítome dos opostos. Se isso não bastasse, observe que cada um dos exemplos de comparação são dadas em sua ordem precisa: sementeira deve anteceder a colheita, em todos os casos. Existia frio antes da criação do sol. Porque o ano bíblico começa na primavera, o verão vai ocorrer pela primeira vez como o ano termina com o inverno. Da mesma forma, o dia começa antes da noite em perfeita harmonia com os três exemplos anteriores. Cada uma destas comparações são pontuadas com o ponto dado a Noé ao sair da arca, que estes devem permanecer perpetuamente na terra e não cessarão. Pergunta 20: É "meia-noite" no meio da noite ou no meio do dia? E assim foi, ao meio-dia, Elias zombava deles e disse: "Clama em alta voz, pois ele é um [Elohiym]; pode ser que esteja meditando, ou ele está ocupado, ou ele está em uma viagem, ou talvez ele está dormindo e deve ser despertado. " 1 Reis 18:27 NVI Resposta: É o meio da "noite", porque a meio do dia vem depois da manhã e antes do por do sol e é chamado de meio-dia. Pergunta 21: Quando as Sagradas Escrituras falam sobre o romper do dia em 2 Samuel 2:32, Juízes 19:25 e Jó 07:04; isso é falar de nascer ou pôr do sol?
  • 20. Então levantaram a Asael eo sepultaram no túmulo de seu pai, que estava em Belém. E Joabe e seus homens caminharam toda aquela noite, e eles vieram até Hebrom ao amanhecer. 2 Samuel 2:32 NTLH Mas os homens não acatam ele. Então, o homem pegou da sua concubina e levou-a para fora para eles. E eles sabiam que ela e abusou dela durante toda a noite até de manhã, e quando o dia começava a quebrar, eles deixá-la ir. Juízes 19:25 NVI Resposta: Ambos estão falando do nascer do sol. Nós podemos saber com certeza que a alvorada se fala aqui é específico ao nascer do sol, porque é claro que essas coisas ocorreram durante toda a noite. O período de crepúsculo é conhecida como a quarta vigília da noite do Daybreak Watch. Próprio Daybreak é sinônimo de o sol nascer. Pergunta 22: Foi o sol feito para governar o dia ou foi feito para governar a noite? Então [Elohiym] disse: "Haja luzeiros no firmamento dos céus para separar o dia da noite, e sejam eles para sinais e para estações, e para dias e anos, e sejam eles para luminares na expansão dos os céus, para alumiar a terra ", e assim foi. Então [Elohiym] fez duas grandes luzes: a maior para governar o dia, eo luminar menor para governar a noite. Ele fez também as estrelas. [Elohiym] colocou-os no firmamento dos céus, para alumiar a terra, e para governar o dia ea noite, e para separar a luz das trevas. E [Elohiym] viu que era bom. Gênesis 1:14-18 NVI Resposta :: Foi feito para governar o dia e não a escuridão da noite. Pergunta 23: Uma vez que o sol foi feito para governar o dia, então como pode um dia começa à noite, quando o sol não está governando ou medir? Àquele que fez os grandes luminares, porque a sua misericórdia dura para sempre, o sol para governar de dia, porque a sua misericórdia dura para sempre, a lua e as estrelas para presidirem a noite, porque a sua misericórdia dura para sempre. Salmos 136:7-9 NVI Resposta: Não pode, isso é impossível porque o Criador projetou esses três tipos de luzes para declarar as divisões específicas de cada data do
  • 21. calendário lunar. Dia e noite simplesmente nunca podem se sobrepor mais do que plena luz ea escuridão completa pode sobrepor-se, como as trevas desaparecem, produzindo apenas plena luz. A escuridão é apenas a ausência de plena luz, assim, portanto, "noite" é a ausência de "dia". Pergunta 24: É declarado várias vezes ao longo das Escrituras, que o Criador dividiu o dia ea noite, ou a luz das trevas (Gênesis 1:3-5, 14- 19). Então, se o dia começa ao pôr do sol, isso levanta a questão, quando foi Ele juntá-los novamente? Nunca. Nada pode ser adicionado a ela, e nada feita a partir dele. [Elohiym] faz isso, que os homens temam diante dele. Eclesiastes 3:14 NTLH A erva seca, a flor murcha, mas a palavra de nosso [Elohiym] permanece para sempre. Isaías 40:8 NTLH Resposta: Ele nunca colocá-los juntos novamente. Muitos optam por acreditar que a luz do dia segue a noite nas Escrituras 24 horas data calendário lunar. Isso é porque eles acreditam que o dia começa ao pôr do sol, mas essa crença não é baseada em peso da Escritura de provas. Pergunta 25: É possível para os meus pais que visitam a permanecer em casa o dia todo, sem passar a noite? Fazes as trevas, e vem a noite, na qual todos os animais da floresta sobre fluência. Salmos 104:20 NKJV Resposta: Sim, porque quando um dia termina, a noite começa. Pergunta 26: Em Números 11:32, diz que os filhos de Israel se reuniram a codorna todo aquele dia e toda aquela noite, e todo o dia seguinte. Então, quantos dias e quantas noites eles se reúnem codorna? E o povo ficou acordado que todos os dias, durante toda a noite e todo o dia seguinte, e colheram as codornizes (aquele que colheu menos, colheu dez home runs), e as estenderam para si ao redor do arraial. Números 11:32 NVI
  • 22. Resposta: Eles se reuniram codorna por dois dias e uma noite, mas se um dia começa à noite, em seguida, eles teriam se reuniram codorna três dias e três noites. Mas a evidência mais profunda no versículo acima é a referência que a noite pertenceu ao primeiro dia mencionado, com a referência do segundo dia mencionado como o "dia seguinte". Pergunta 27: Se o dia começa ao nascer do sol, então por que os versículos bíblicos que descrevem quando o Salvador ressuscitou, diga "de manhã cedo", enquanto ainda estava escuro? Resposta: Ótima pergunta. Um erro foi descoberto nas traduções que colocam a palavra "cedo" ou "início da manhã" no lugar da palavra grega "proi" 4404 #. Embora tenha sido regularmente traduzido para significar "no início da manhã", que na verdade significa que durante a "quarta vigília da noite" ou "Daybreak Watch" ou Ver Marcos 16:2,
  • 23. 9 "de madrugada.", João 20:01; Mateus 20:01
  • 24. Concordância de Strong Definição grego: # 4404 (proi) = ao amanhecer; por implicação, o DaybreakWatch. 1. Proi é a quarta vigília da noite. Por favor, não perca este ponto. "Amanhecer e entardecer" são ambos equacionada com a "noite" e não com o "dia". É nestes versículos bíblicos muito destinados a provar que o amanhecer faz parte do dia em que o peso da evidência é dada pelo contrário. Dawn é sinônimo com a última vigília da noite. Estes versos incluem todas as referências de tempo no Novo Testamento para a Ressurreição. O texto grego esclarece que a ressurreição realmente aconteceu no "primeiro" da semana, que foi também a data do calendário lunar 16. Mas o que é mais surpreendente, é que a ressurreição ocorreu no final da data do calendário lunar do dia 16, durante a quarta vigília da noite. Devido a esta descoberta de como data do calendário lunar do Criador é dividido, agora é possível contabilizar o Messias estar no túmulo por três dias e três noites. Em breve - Consulte o artigo na íntegra, "três dias e três noites, o sinal profético de Jonas." Pergunta 28: Se Yahuah deu a celeste maná somente com a finalidade de alimentar o povo e ilustrando que dia foi o sétimo dia de sábado , então por que há tanta escrito em Êxodo 16 sobre outras unidades de tempo definidos pelo maná diariamente? Resposta: Em ternos cuidados do Pai Celestial, Ele fez chover maná do céu durante quarenta anos no deserto para alimentar os israelitas recém-lançado quem levaram para fora do Egito. Além de esclarecer que dia foi o sábado do sétimo dia, existem vários outros detalhes centric tempo relacionadas com o maná de grande importância para Yahuah, que ilustram quando o dia começa. Ele deu especificações precisas sobre a reunião do maná como Ele afirmou que seria usado para testar a sua obediência a ele. Era Seu plano ordenado de usar uma programação de alimentação para restaurar Israel de volta para Seu sistema tempo lunissolar.
  • 25. . . . Que eu possa prová-los, se anda em minha lei ou não. Êxodo 16:04 . . . as pessoas devem sair e reunir o dever de cada dia. . . Êxodo 16:04 . . . no sexto dia, que prepararão o que colherem; e será o dobro do que colhem cada dia. Exodus16: 5 . . . Pela manhã, vereis a glória de [Yahuah]. . . Êxodo 16:07 pela manhã vos fartareis de pão, e sabereis que eu sou [Yahuah] seu [Elohiym]. Êxodo 16:12 E, quando o orvalho que estava se levantou, eis que sobre a face do deserto estava uma coisa miúda, redonda, miúda como a geada sobre a terra. Êxodo 16:14 Ninguém deixe dele para amanhã. Exodus16: 19 . . . alguns deles deixaram dele para a manhã [próximo], e criou bichos, e cheirava mal. . . Êxodo 16:20 . . . quando o sol se acendeu, derretia-se. Êxodo 16:21 . . . no sexto dia eles se reuniram duas vezes mais. . . Êxodo 16:22 Amanhã é repouso, sábado santo [Yahuah]: coza o que vos vai assar hoje, e faze que vos vai ferver, eo que permanece sobre ajuntai para que você possa ser mantido até a manhã. Êxodo 16:23 E guardaram-no até o dia seguinte, como Moisés tinha ordenado, e não cheirou mal, nem houve nele bicho algum. Êxodo 16:24 E disse Moisés: Comei-o hoje, porquanto hoje é o sábado para o [Yahuah]:-a-dia não o achareis no campo. Êxodo 16:25 Seis dias o colhereis, mas o sétimo dia, que é o sábado, nele não haverá. Êxodo 16:26 Veja, por que [Yahuah] vos deu o sábado, por isso ele lhe dá no sexto dia o pão para dois dias; cada um fique no seu lugar, não saia ninguém do seu lugar no sétimo dia. Êxodo 16:29 Assim repousou o povo no sétimo dia. Êxodo 16:30
  • 26. Aqui verifica-se que o sábado do Pai Celestial é o sétimo dia e que segue seis dias de trabalho. Verificou-se que começa o dia de manhã, em vez de durante a noite. O maná poderia ser preservado para exatamente 24 horas a partir do momento em que foi dado, e sempre foi ruim na manhã seguinte, quando o fresco, novo maná foi fornecido, exceto no sábado. Nós descobrimos que o termo hoje é referido como um dia, que se inicia no dia. No sexto dia, Moisés falou de amanhã como sendo o dia de sábado. Se o sábado começou ao pôr do sol, Moisés teria falado a respeito da noite de sábado ou ao pôr do sol. O sábado começou na parte da manhã, como descrito por nenhum maná dado naquela manhã. Em todo este Êxodo 16 narrativa não há uma menção da palavra noite, pôr do sol, crepúsculo ou à noite. O peso da evidência declara nesta história só que o dia começa na manhã seguinte. Pergunta 29: Desde o tempo de Moisés, até o morte do Salvador na cruz, um período de 1500 anos, tanto pela manhã e sacrifícios noite foram uma ocorrência diária. Era só o sacrifício da manhã que foi administrada durante o dia, com os sacrifícios da noite sendo administrado durante a escuridão da noite? Resposta: Não! Ambos os sacrifícios foram administradas durante o dia. O sacrifício da manhã foi administrada logo após o nascer do sol, enquanto o sacrifício da tarde foi administrada pouco antes do por do sol. De acordo com escrituras, a noite foi igualado com a troca de luz que ocorreu ao amanhecer e ao pôr do sol novamente. Este é um conceito novo para a maioria, mas traz todos os pontos nas Escrituras em harmonia, sobre este assunto de quando um dia começa. (Êxodo 29:39). Pergunta 30: De acordo com Exodus18: 13, quando é que "no dia seguinte" começar, à noite, a noite ou de manhã? E assim foi, no dia seguinte, Moisés assentou-se para julgar o povo, eo povo estava em pé diante de Moisés desde a manhã até a noite. Exodus18: 13 NVI
  • 27. Resposta: O "dia seguinte", começa na parte da manhã, como todos das Escrituras indicam claramente. Você nunca vai encontrar a Escritura declara que o "dia seguinte", ou amanhã começa à noite ou a noite, mas sempre começa ao nascer do sol. Pergunta 31: De acordo com Êxodo 16:22-25, quando é que o resto do sábado santo Yahuah começar, no meio da noite, à noite ou de manhã? E assim foi, no sexto dia, que eles se reuniram pão em dobro, dois omers para cada um. E todos os príncipes da congregação vieram, e contaram Moisés. Então ele lhes disse: "Isto é o que [Yahuah] disse: 'Amanhã é um descanso sabático, sábado santo ao [Yahuah] Asse o que você vai assar hoje, e ferva o que você vai ferver, e ajuntai para vós. tudo o que resta, para ser guardado até a manhã ". "Então eles guardaram-no até de manhã, como Moisés tinha ordenado, e não cheirou mal, nem houve quaisquer vermes nele. Então Moisés disse: "Comei-o hoje, porquanto hoje é o sábado [Yahuah], hoje você não vai encontrá-lo no campo Êxodo 16:22-25 NTLH. Resposta: Observe que a referência feita por [Yahuah] foi feito no dia da preparação, declarando que amanhã é o descanso sabático. O sábado começa na parte da manhã como todos os outros dias começar. Então, quando naquela manhã seguinte chegou, temos uma declaração de confirmação de Moisés que disse: "Comei-o hoje, porquanto hoje é o sábado [Yahuah]." Isso deixa claro que o sábado começou no dia seguinte ao amanhecer. Pergunta 32: Se uma noite é considerado um dia, então o que Yahuah dizer quando disse que iria chover sobre a terra quarenta dias e quarenta noites? Para depois de mais de sete dias farei chover sobre a terra quarenta dias e quarenta noites, e eu o destruirei de sobre a face da terra todos os seres vivos que eu fiz. Gênesis 07:04 NVI Resposta: A noite não é considerado o dia, ou até mesmo o início de um dia, mas, em vez dias e noites são opostos exactos. Além disso, o aviso prévio, nas palavras de Yahuah Si mesmo, Ele afirma que o "dia" vem antes do "noite", quando disse que choveria quarenta dias e quarenta
  • 28. noites. O Pai Celestial sempre faz declarações precisas, e Ele não disse que chover- Pergunta 33: Por que é que os homens justos são despertou contra os hipócritas? O que as pessoas fazem para merecer ser chamado de hipócritas? Os retos estão surpresos com isso, e os inocentes agita-se contra o hipócrita. No entanto, o justo irá realizar para o seu caminho, e ele que tem as mãos limpas será mais forte e mais forte. "Mas, por favor, volte mais uma vez, a todos vocês, pois eu não encontrar um homem sábio entre vós, meus dias passaram, meus propósitos são quebrados, mesmo os pensamentos do meu coração Eles mudam a noite em dia;. '. A luz está perto ', dizem eles, em face da escuridão. Job 17:8-14 NVI Resposta :: Porque os hipócritas estão dizendo que Yahuah, nosso Criador, é um mentiroso quando ele disse que separou o dia da noite. Gênesis 1:04. Agora eles mudaram a noite em dia, quando Ele havia declarado a luz para ser o dia e não a escuridão da noite. Pergunta 34: Se o dia foi destinado desde a criação até início da noite, então como alguém poderia mudar a noite em dia, tal como indicado neste versículo, se ele já foi? E a palavra do [Yahuah] veio a Jeremias, dizendo: "Assim diz [Yahuah]:" Se você pode quebrar a minha aliança com o dia eo meu pacto com a noite, de modo que não haverá dia e noite a seu tempo, em seguida, a minha aliança também pode ser quebrado com Davi, meu servo, de modo que ele não terá um filho que reine no seu trono, e com os levitas, sacerdotes, meus ministros. Jeremias 33:19-21 NTLH Resposta: Isso seria totalmente impossível. Pelos maus israelitas que tentam fazê-lo, provocaria a destruição total da casa de Israel. Pergunta 35: De acordo com as palavras do Altíssimo, que é a diferença entre um dia e uma noite? Assim diz [Yahuah], que dá o sol para luz do dia, as ordenanças da lua e das estrelas para luz da noite, que agita o mar e suas ondas rujam ([Yahuah] dos Exércitos é o seu nome): "Se estas ordenanças de diante
  • 29. de mim, diz [Yahuah], então a semente de Israel deixam também de ser uma nação diante de mim para sempre." Jeremias 31:35-36 NTLH Àquele que fez os grandes luminares, porque a sua misericórdia dura para sempre - o sol para governar de dia, porque a sua misericórdia dura para sempre, a lua e as estrelas para presidirem a noite, porque a sua misericórdia dura para sempre. Salmos 136:7-9 NVI Resposta: Day é igual a luz solar, e de noite é igual ao período com a lua e as estrelas. Jeremias 31:35 e Salmo 136:7-9. Gênesis 1:05 diz: "[Elohiym] chamou à luz dia, e às trevas chamou noite. Pergunta 36: Em Jeremias 33:19-21, Yahuah fala sobre o mal que Ele traria sobre a casa de Davi e do reino de Israel se eles quebraram seus convênios. Qual foi o pacto que Ele foi sublinhando que o seu povo não quebrar? Assim diz [Yahuah], que dá o sol para luz do dia, as ordenanças da lua e das estrelas para luz da noite, que agita o mar e suas ondas rujam ([Yahuah] dos Exércitos é o seu nome): "Se estas ordenanças de diante de mim, diz [Yahuah], então a semente de Israel deixam também de ser uma nação diante de mim para sempre." Jeremias 31:35-36 NTLH Resposta: Isso está se referindo especificamente aos Seus preceitos do "sol" para luz do dia, e as ordenanças da "lua e as estrelas" para uma luz da noite. Isto é revelar Sua pedaço de tempo específico ordenado lunar-solar celestial. Se essas divisões ordenados de tempo são quebradas ou modificadas que causaria a 12 horas "dia", e os quatro relógios da "noite", não estar em suas posições de tempo divinamente designado. Este versículo tem uma maior aplicação do que aparece pela primeira vez na superfície. Ele está afirmando que, se as ordenanças de "sol" para dias e "lua e as estrelas" para as noites sai, assim será a "semente de Israel" cessar de ser uma nação para sempre. "Em outras palavras, este é um confirmado prometida de Yahuah que Ele nunca vai mudar seu sistema de tempo astro-luni-solar por Deus. Isso é garantido no fato de que, "se você é [o Messias], também sois descendentes de Abraão, e herdeiros conforme a promessa." Gálatas 3 :28-29. O ponto aqui é que haverá uma segunda vinda do Messias para os remidos do
  • 30. Terra. Todos estes remidos do terra, desde o tempo de Abraão, são ou judeu ou gentio, mas se ser o Messias de todos eles são Israel espiritual, a semente de Abraão e será recompensado com a vida eterna. Portanto, a garantia é que a semente fiel de Abraão nunca cessará, assim como as ordenanças do sol, a lua e as estrelas que representam as divisões do "dia "e" noite "deve nunca cessar. Por outro lado, logo depois que os planos da crucificação foram estabelecidas pelos líderes judeus para mudar calendation de Israel. A Lua Nova foi alterado a partir da "lua cheia" para o "primeiro crescente visível." O ciclo semanal romano foi adotado para o sábado do sétimo dia, eo "dia" foi alterado para começar ao pôr do sol, em vez de nascer do sol. Vai ser todos aqueles que continuam a andar no tempo artificial dos judeus e romanos, que deixará de ser uma nação para sempre, de acordo com o versículo acima. Daniel 7:25 esclarece que os tempos e as leis de Yahuah seriam adulterados pelo homem. Pergunta 37: O que [Yahuah] declarar que acontecerá se o decreto-lei de dia e à ordenação de noite são removidos? Assim diz [Yahuah], que dá o sol para luz do dia, as ordenanças da lua e das estrelas para luz da noite, que agita o mar e suas ondas rujam ([Yahuah] dos Exércitos é o seu nome): "Se estas ordenanças de diante de mim, diz [Yahuah], então a semente de Israel deixam também de ser uma nação diante de mim para sempre." Jeremias 31:35-36 NTLH Resposta: Israel deixará de ser uma nação. O foco é em Israel pela simples razão de que tinha sido Israel que teve a honra de manter o sistema de tempo correto Yahuah como um farol de verdade sempre antes de as nações da terra. Poderia ser que a reforma cronometragem surgiu após a crucificação do Messias, como os líderes perceberam o serviço do Templo e seu modelo de calendário astro-luni-solar continuamente lembrou-os de sua culpa. Pois foi tanto os serviços do Templo e do calendário que prefigurou toda a obra do Messias. Esta mudança na calendation, por líderes de Israel e Roma foi profetizado para ocorrer por Daniel o Profeta (Daniel 7:25).
  • 31. Pergunta 38: Será que não Levítico 23:32 suporte o "dia" do início ao pôr do sol e termina ao pôr do sol, como ele afirma, "do mesmo até mesmo você deve celebrar seus sábados?" Resposta: Muitas sabatistas têm usado Levítico 23:32 como uma segunda testemunha de Gênesis 1:5, declarando que devemos santificar o sábado do sétimo dia de "até a mesmo." Eles interpretam "do mesmo até mesmo", como "a partir de pôr do sol até pôr do sol. " Enquanto isto parece provável, à primeira vista, a verdade pura e não adulterada está dentro do texto original hebraico e não é encontrada nas modernas versões traduzidas como ilustrado abaixo. Ele vos será o sábado de descanso solene. . . desde a tarde até a noite, você deve celebrar o seu sábado. "Levítico 23:32 NVI Este versículo está declarando os parâmetros da festa "Dia da Expiação", que é anual "dia de festa" que ocorrem a cada ano. É, simultaneamente, fixado ao "décimo data" da contagem mês a partir da "cheia" dia de Lua Nova, e para o segundo dia da semana lunar. Isso é verificado no fato de que Festa dos Tabernáculos é no dia 15 do mês, e consistentemente fixado ao sábado do sétimo dia. Este versículo em Levítico 23:32 não está se referindo especificamente ao sábado do sétimo dia não mais do que a descrição da Festa da Páscoa no versículo 5. No entanto, parece que os parâmetros de tempo para esta festa, "Dia da Expiação", que também se aplicam ao sábado do sétimo dia.
  • 32. De acordo com a Escritura amanhecer e entardecer são equiparados com o primeiro eo quarto vigílias da noite. O crepúsculo amanhecer é chamado de manhã ou Daybreak Watch, enquanto o crepúsculo anoitecer é chamado de Noite Watch. A palavra que é traduzida em Levítico 23:32, para ser "mesmo", é a mesma palavra que é traduzida em verso Levítico 23:05 como "Crepúsculo". A palavra hebraica pode ser derivado tanto # 6153 `Ereb ou # 6154 como` Ereb, pois estas são as mesmas palavras hebraicas exatas com definições diferentes.
  • 33. Strong # King James Version da Bíblia Hebraico Strong Definições Brown-driver-Briggs Hebrew Lexicon 6153 a noite `Ereb `Ereb # 6154 - Palavra - Root # 6148 `árabe Palavra Raiz - # 6150 `árabe anoitecer; maré da noite, noite, dia mistura trançar, intermix; se misturam; dar penhores; envolver, dar para ser de segurança (como uma espécie detroca); através da idéia de cobrir com textura; sol; ser escurecido; pôr do sol, noite,noite; # 6154 `Ereb - que é a mesma palavra hebraica: trama, como misto;entrelaçados; material de malha; mistura; pessoas mistos; sociedade de economia mista. a troca, para envolver, dar em penhor, para assumir um compromisso, para garantir; para troca de promessas; ter comunhão com; O segredo para desvendar este mistério é em hebraico a palavra # 6154 `Ereb, o que significa mistura de luz ou crepúsculo. Dicionário Definição de Webster:
  • 34. crepúsculo = 1: a luz do céu entre a noite eo nascer do sol pleno ou entre o pôr e noite completo, produzido pela difusão da luz solar através da atmosfera e sua poeira. Curiosamente, não há apenas um, mas dois períodos de crepúsculo conectado com cada data do calendário 24 horas. Em primeiro lugar, há o crepúsculo que precede nascer do sol, bem como o crepúsculo que segue pôr do sol. Mas, mais específico ainda, é nesse período culmina em uma troca de luz absoluta. Estes incluem o período das trevas para a completa troca radiante luz do sol ao nascer do sol, ou a luz do sol radiante para troca escuridão ao pôr do sol (à noite). Já foi estabelecido que o Criador está interessado apenas em porções de luz do dia como os raios radiantes de luz solar simbolizam verdade e da justiça. Portanto, a frase "desde a tarde até mesmo está realmente se referindo ao crepúsculo anterior e seguindo as horas do dia, comportando-se como suportes para a parte sagrada deste dia de festa da Expiação. Pensar fora da caixa, é possível, em qualquer nível que a frase "desde a tarde até mesmo" foi destinado por Moisés para significar "a partir de crepúsculo até o crepúsculo", as trocas de luz que ocorrem tanto o nascer eo pôr do sol? Isto parece mais provável. Com isto em mente, vamos voltar a Levítico 23:32, é de todo possível que Moisés é esclarecer "de crepúsculo até o crepúsculo, você deve comemorar seu sábado na data lunar décimo? Poderia estar se referindo ao nascer e pôr do sol troca de luz ? Este é um período de 12 horas de tempo e não pode incluir um 24 horas data calendário lunar cheia de uma revolução solar completa. Como resultado "do mesmo até mesmo," seria sinônimo de "de sol a sol", uma mera 12 horas período. Se isso for verdade, então "até mesmo (nascer do sol) até mesmo (por do sol)" só seria a parcela de tempo entre o ponto A eo ponto B. Estes
  • 35. estão definindo os parâmetros de apenas luz do dia, começando ao nascer do sol e termina ao pôr do sol. Uma vez que esta porção de tempo definido é um período entre dois "trocas de luz," só há duas opções a respeito de quando esse espaço de tempo é. Teoricamente, poderia ser do pôr ao nascer do sol, que engloba apenas o escuro da noite, ou de sol a sol abrangendo apenas a luz do dia. Mas, nós temos isso na boa autoridade de Moisés e Yahuah que esta parcela 12 horas de uma data do calendário lunar era de fato do nascer ao pôr do sol, pois o simples fato de que esta era a terminologia usada para descrever a "luz do dia", como é chamado " Dia "da Expiação e não a" Noite "da Expiação. Consulte o estudo da palavra, Gênesis 1:1-5 "Quando um dia começa." Pergunta 39: Gênesis, Êxodo, Levítico e foram os três primeiros livros escritos por Moisés. Se os elementos de tempo esclarecidas em cada um desses livros não estavam em perfeita harmonia uns com os outros, Moisés teria questionado Yahuah. Foi Moisés quem gravou os Dez Mandamentos, inclusive o quarto, que fala especificamente do sábado do sétimo dia. Existe alguma referência na Bíblia para iniciar o sábado à noite? Resposta: Na Bíblia, não há nenhuma referência ao sábado início da noite ou do sol, além dos erros de tradutor de Gênesis 1 e Levítico 23:32, como mencionei anteriormente. O quarto mandamento diz claramente que o Sabbath era ocupar o período de tempo conhecido como a luz do dia e qualquer referência é feita para a escuridão da noite. Lembre-se do dia de sábado, para santificá-lo. Seis dias trabalharás, e farás toda a tua obra, mas o sétimo dia é o sábado [Yahuah] seu [Elohiym]. Nele você deve fazer nenhum trabalho: você, nem o teu filho, nem tua filha, nem o teu servo, nem a tua serva, nem o teu animal, nem o estrangeiro que está dentro das tuas portas. Porque em seis dias [Yahuah] fez os céus ea terra, o mar e tudo o que neles há, e ao sétimo dia descansou. Portanto [Yahuah] abençoou o dia do sábado, eo santificou. Êxodo 20:8-11 NVI
  • 36. No entanto, alguns gostariam de nos fazer pensar que, após a crucificação de Yahushua as pernas dos dois criminosos foram quebradas e que, juntamente com o Messias, foram removidas de suas cruzes, porque o "sétimo dia" foi com base na em três horas. No entanto, nós sabemos para um fato que a Páscoa era para começar em três horas como era para começar ao pôr do sol. O sábado imediato que estava chegando na festa da Páscoa era sábado, o que era para começar precisamente ao pôr do sol. Todos os homens judeus juntos com suas famílias, eram para estar em casa para o evento e foi-lhes ordenado "para não pisar fora da porta de suas casas até a manhã" (Êxodo 12:22). O sábado do sétimo dia era para começar na manhã seguinte ao nascer do sol, no momento exato da Páscoa terminou. O sábado do sétimo dia nunca é mencionado nas Escrituras como a sobreposição, mas era sempre o dia seguinte da Páscoa e começou ao amanhecer. Era a festa de sete dias dos pães ázimos, que sempre começou no sábado do sétimo dia, tornando-se uma alta sábado. Mark 15:42-44 Agora, quando já era tarde, porque era a preparação, isto é, a véspera do sábado, José de Arimatéia, um membro do conselho de destaque, que também esperava o reino de [Elohiym], indo e tomando coragem, foi ter com Pilatos e pediu o corpo de Jesus. A referência a "quando já era tarde" é claramente indicado para ser um período de tempo antes do início do sábado. Era a continuação da preparação, mas foi o início da Páscoa que já havia começado ao pôr do sol. . . A palavra "dia" não existe no texto original. A palavra # 4315 prosabbaton simplesmente significa antes do sábado. O outro identificador neste versículo declara que o sol já se pôs, como este é o significado da palavra # 3798 opsias, o que significa o cair da noite. Observe como esse novo testamento a palavra # 3798 noite de outono está em harmonia com o Antigo Testamento # 6153 Ereb que significa crepúsculo. É evidente que o sábado ainda estava para começar na manhã seguinte junto com a Festa dos Pães Ázimos. Era a festa da Páscoa, que tinha acabado de começar ao pôr do sol no dia 14. Agora era o dia da preparação da Páscoa, e cerca da hora sexta. E disse aos judeus. . . João 19:14
  • 37. Agora, no lugar onde Ele foi crucificado havia um jardim, e no jardim um sepulcro novo, em que ninguém ainda havia sido posto. Portanto, não puseram a Jesus, por causa da preparação dos judeus ', para o túmulo estava perto. João 19:41-42 A preparação aqui mencionado foi o sexto dia da semana, porque o dia seguinte era uma alta sábado. Mas a preparação urgente aqui é duplo. Aqui, novamente, a palavra "dia" foi acrescentada pelos tradutores. O primeiro é a preparação para a Festa da Páscoa, que começou ao pôr do sol do dia 14. A Festa da Páscoa dura do pôr ao nascer do sol na manhã seguinte do dia 15. O sábado do sétimo dia não era para começar até o amanhecer do dia seguinte, altura em que a Festa dos Pães Ázimos começou bem. Alta sábado era o termo usado quando uma festa sábado anual nomeado pousou em um sábado do sétimo dia regular. Isso ocorre apenas duas vezes por ano, e neste caso, foi o primeiro dia da Festa dos Pães Ázimos. Além disso, observe a última frase. Ele abençoou o dia de sábado eo separou como sagrado e santo. Páscoa é o único lugar na Bíblia onde o escuro da noite é designado como A luz era para simbolizar a verdade ea justiça. Não há inconsistências em relação a este simbolismo ordenado. O sábado era para ser apenas as horas do dia. Era para ser de crepúsculo até o crepúsculo, que é o verdadeiro significado de "a partir mesmo até mesmo". Levítico 23:05, 32. Abaixo está um relógio de sol grego antigo, que, por sua concepção está em harmonia com as Escrituras (João 11:9). Embora este relógio de sol está faltando seu "gnomon" (ponteiro) observar os números são da esquerda para a direita, começando no número 1 e terminando no número 12. Isso demonstra que a primeira hora após o nascer do sol é a primeira hora do dia. Ele também demonstra que a 6 ª hora do dia é meio-dia, e 12 ª hora termina precisamente pôr do sol. Se é verão ou inverno sempre há 12 horas em um dia. Relógios de sol só medir raios diretos do sol, e não medem a noite, ou qualquer parte do anoitecer ou de madrugada. É o projeto original do relógio que determina os parâmetros mensuráveis de um dia, separando o dia da noite. Em
  • 38. contraste marcante, os relógios de sol gregorianos romanas seguir um sistema numérico que exibe a 12 ª hora, como o meio-dia, correlacionando-se com um dia de 24 horas que começa à meia-noite. De acordo com a Escritura, do meio-dia era a sexta hora do dia. Alfabeto grego e do sistema numérico:
  • 39. Pergunta 40: Durante a vida do Messias Ele guardou o sábado para o comprimento correto de tempo. Se Israel foi mantê-lo errado, certamente ele teria corrigido eles. Quanto tempo durou o Messias declarar um "dia" de ser, o que também se aplica ao dia do sábado do sétimo dia? Jesus respondeu: "Não são doze as horas do dia? Se alguém andar de dia, não tropeça, porque vê a luz deste mundo." João 11:09 NVI Resposta: Enquanto Ele afirma que o dia seja 12 horas de duração, observe Ele esclarece nenhuma diferença entre o comprimento de um dia da semana e um dia de sábado. Desta forma, Ele os define como o mesmo. Os judeus tradicionais não são capazes de utilizar esse detalhe, porque a não reconhecer ou o Messias ou o Novo Testamento.ia quarenta noites e quarenta dias. Pergunta 41: Além disso, durante a vida do Messias que era seu costume ir à sinagoga. Esteve presente durante a parte clara ou escura de uma data do calendário lunar? E, como seu costume, ele entrou na sinagoga no dia de sábado, e levantou-se para ler. Lucas 4:16 NTLH Resposta: De acordo com a Escritura que era seu costume de entrar na sinagoga durante a luz do dia. No onde está registrado que ele foi à sinagoga para o culto durante a escuridão da noite. Os judeus tradicionais não são capazes de utilizar esse detalhe. Pergunta 42: O Messias, Yahushua era o Cordeiro sacrificial que deu a sua vida como oferta pelo pecado, em nome da humanidade. Depois de um julgamento de agonia e espancamento, nosso Salvador foi colocado
  • 40. na cruz na sexta hora. Isso levanta a questão, foi esta sexta hora do dia, ou a sexta hora da noite? Agora era o dia da preparação da Páscoa, e cerca da hora sexta. João 19:14 NVI Resposta: Quando este afirma que era a "hora sexta", então você deve perceber que este tempo contínuo é contado a partir do nascer do sol, fazendo nascer no início de cada novo dia. Como resultado, o Salvador teria sido colocado na cruz, às 12h00. No entanto, se você insistir o dia começa na "noite", porque você acredita que o "dia" começou a contar a partir do sol, então você também deve acreditar que o Salvador foi crucificado em cerca de 12:00 à meia-noite e morreu por volta de 03h00 (Gregoriano tempo), durante a escuridão da noite. Você não pode ter as duas coisas. Como um prego num lugar firme, um eclipse solar ocorreu no dia da crucificação, do sexto ao nono hora. Os eclipses solares ocorrem apenas durante o dia, quando o sol é totalmente visível. Veja gráfico:
  • 41. Pergunta 43: Páscoa, Abib 14, AD 31, foi a data do calendário lunar da crucificação. Este dia de dias, também conhecido como o dia da preparação, continha apenas 12 horas de sol a sol. Quantas horas do dia seguinte contém? Agora era o dia da preparação da Páscoa, e cerca da hora sexta. João 19:14 NVI Agora, a partir da sexta hora até a hora nona, houve trevas sobre toda a terra. Mateus 27:45-46 Agora, quando a sexta hora tinha chegado, houve trevas sobre toda a terra até a hora nona. Mark 15:33-34
  • 42. O Salvador foi colocado na cruz na sexta hora do dia. No momento da sexta hora (meio-dia) havia trevas sobre toda a terra até a hora nona (3:00 PM gregoriano Time). Lucas 23:45 esclarece, ainda, que isso foi causado pelo sol escurecido. Um sol escurecerá é sinônimo de um eclipse solar. Para 4000 anos todos os eclipses solares ocorreu no dia 14 ou 15 do mês lunar, e este mês não foi excepção ao sistema de tempo do Criador. Pelo contrário, era o cumprimento de todas as maneiras, e com destaque para um comprimento sobrenatural das trevas, como a atenção de todo o céu estava voltada para a Terra e do Filho morrendo de Yahuah. Consulte o estudo da palavra, "O sol escureceu da sexta para a Nona Hora". Resposta: O dia seguinte continha exatamente a mesma quantidade de horas, o que foi um total de doze. O dia seguinte à crucificação, não era apenas um sábado do sétimo dia, mas também foi o primeiro dia da Festa dos Pães Ázimos, tornando-se uma alta sábado. O número de horas sagradas para este dia era exatamente o mesmo que as horas sagradas do sétimo dia da criação. A parte sagrada do sábado do sétimo dia tem sido sempre apenas as horas do dia, e nunca foi para incluir a escuridão da noite. Os judeus tradicionais não são capazes de utilizar esse detalhe. Pergunta 44: "nascer do sol, até o seu ocaso" Se a celebração do nosso dia de sábado começa quando o sol está se pondo, por que o nosso estado Criador que todas as nações louvarão o Seu nome e adorá-lo a partir do Desde o nascer do sol até o seu ocaso nome de Yahuah é para ser elogiado. Salmos 113:3 NVI O Poderoso, [Elohiym] [Yahuah], falou e chamou a terra desde o nascer do sol até o seu ocaso. Desde Sião, a perfeição da beleza, [Elohiym] brilhará. Salmos 50:1-2 NVI Mas desde o nascente do sol, até o seu ocaso, meu nome é grande entre os gentios, em todo lugar incenso deve ser oferecido ao meu nome, e uma oblação pura; porque o meu nome é grande entre as nações ", diz [Yahuah] dos Exércitos. Malaquias 1:11 NTLH
  • 43. Resposta: Se o dia de sábado começa ao pôr do sol ou na primeira luz da aurora, como muitas pessoas acreditam, então estes versos não faria sentido algum. A Escritura define claramente que o louvor do Todo- Poderoso [Elohiym] do Céu é feito de sol a sol. Recap: A primeira coisa que o Criador chamou à existência era "luz". Gênesis 1:3.A escuridão não foi criado pelo Criador Yahuah, mas sim que é a ausência de luz e Seu poder criativo. Gênesis 1:2-4. Um dia é a parte "luz do sol" de uma data do calendário lunar total, ocupando o período de sol a sol e é definido pelo número de horas em um relógio de sol. Gênesis 1:4, 5. A noite ocupa a porção do pôr ao nascer do sol, de uma data do calendário lunar completo, e é definida por quatro relógios. Gênesis 1:4-5. Dia e noite são períodos opostos do tempo que nunca se sobrepõem. Gênesis 1:4, 5. Uma distinção que separa estava sempre lançar quando o Criador chamado a luz do dia ea escuridão da noite. Gênesis 1:4-5. Dia e noite são as duas metades de uma data do calendário lunar completo. Gênesis 1:4, 5. Noite é equiparada com a mistura de luz conhecido como "Twilight" ou "troca de luz", que ocorre pouco antes do nascer do sol e logo após o pôr do sol. Gênesis 1:05. "O dia começa ao nascer do sol e termina ao pôr do sol e totaliza um período de doze horas. João 11:09. Toda a criação foi realizada durante as horas de luz do dia. exceto quando a luz foi chamado diante das trevas no primeiro dia. Gênesis 1: todos.
  • 44. As quatro vigílias da noite ocupam o período entre o pôr eo nascer do sol, como a duração da noite é definida pelas estrelas visíveis. Jeremias 31:35-36 As regras visíveis sol e as medidas a dia, enquanto as estrelas visíveis governar e medir a noite. O dia ea noite nunca pode sobrepor-se, mais do que o brilho do sol e as estrelas podem se sobrepor ou ser visíveis ao mesmo tempo. Jeremias 31:35-36 O Criador declarou seis vezes em seis dias que Suas obras de criação foram realizadas durante o dia, que foram compostas da noite seguinte pela manhã. Gênesis 1:1-31. O sol foi nomeado para governar o dia. Gênesis 1:16. A lua e as estrelas foram nomeados para governar a noite. Gênesis 1:16. Yahuah sempre usou os princípios contrastantes de luz e escuridão para ensinar suas verdades espirituais. Provérbios 4:18-19, João 8:12, Mateus 5:14, 15; João 1:4, 7; João 3:20, 1 Pedro 2:09, Efésios 5:13. Metaforicamente, "escuridão" foi para representar a infidelidade, a cegueira espiritual e do mal. Provérbios 4:18-19, João 1:04, João 3:20; 1 Pedro 2:9. Metaforicamente, "luz" foi para representar verdade, compreensão espiritual e justiça. Provérbios 4:18-19, João 1:04, João 3:20; 1 Pedro 2:9. Os termos de dia ou amanhã seguinte, ambos referem-se à luz do dia, que começa ao nascer do sol e termina ao pôr do sol. Êxodo 18:13. Hipócritas chamar o dia da noite. Isso é demonstrado por todos aqueles que acreditam que o santo sábado do sétimo dia começa ao pôr do sol. Job 17:8-14. Se o decreto do dia e portaria da noite são removidos, então Israel devia cessar como uma nação. Job 17:8-14.
  • 45. "Os termos" mesmo até mesmo "se referem ao nascer ao pôr como" mesmo "representa a" troca de luz ", e foi criada como suportes de livros metafóricos para um" dia ". Levítico 23:05, 32. O Quarto Mandamento suporta o que foi descoberto em Gênesis 1:1-5, que o sábado é a porção à luz do dia na data do calendário lunar completo. Não há absolutamente nenhuma menção à escuridão da noite no quarto mandamento. Consulte o estudo da palavra, Êxodo 20:7-11 " A Lua Nova Beacon Encontrado no Terceiro e Quarto Mandamento ". O Criador abençoou o dia de sábado eo separou como sagrado e santo. Em nenhum lugar na Bíblia é a escuridão da noite já designado como sagrado e santo do que outro, possivelmente, a Festa da Páscoa, pois só naquela noite foi divinamente projetado para simbolizar a cegueira espiritual e do mal. No caso da Páscoa, era o sangue do Cordeiro de [Elohiym] que era resgatar a humanidade pecadora e as trevas do mundo, caiu em pecado. Provérbios 4:18-19, João 1:04, João 3:20; 1 Pedro 2:9. O dia de sábado é o mesmo período de tempo, como todos os outros dias, incorporando um período de 12 horas de luz do dia. Gênesis 1:1- 5; Gênesis 2:1-3, João 11:09. Durante a vida do Messias que era seu costume de entrar na sinagoga, no "dia" de sábado e nunca a "noite". Lucas 4:16. Durante a vida do Messias Ele declarou que há apenas "12 horas" em um dia. Afinal de contas, Ele também foi o criador, que chamou o "dia" de luz e às trevas chamou "noite". Ele não muda. João 11:09; Eclesiastes 3:14, Isaías 40:8. Em Sua morte Ele confirmou que o dia está apenas doze horas de duração. Ele foi crucificado na terceira hora do "dia". O céu ficou
  • 46. escuro desde a hora sexta (meio-dia) até a hora nona, Ele declarou, está consumado e morreu. Lembre-se da noite não é dividido por horas, portanto isso está se referindo especificamente para as horas do dia. Yahuah declarou que as nações dos gentios da terra iria adorá-lo desde o nascer do sol para ele vai para baixo. Salmos 113:3, Salmos 50:1-2; Malaquias 1:11. Que você seja abundantemente abençoado em tudo que você faz, como você continuar olhando para cima e buscando o Pai das luzes (Tiago 1:17). - Fonte: Kerry Frances http://www.thecreatorscalendar.com/Articles/When_does_a_day_begi n/07_When_does_a_day_begin.html